You are on page 1of 62

IIT-JEE|AIEEE

CBSE|SAT|NTSE
OLYMPIADS

Nurturing potential through education

ELECTROMAGNETIC INDUCTION
THEORY AND EXERCISE BOOKLET

CONTENTS

S.NO. TOPIC PAGE NO.

1. Magnetic Flux ................................................................................................... 3

2. Faraday’s Law of Electromagnetic .................................................................. 3 – 4

Induction

3. Lenz’s Law ..................................................................................................... 4 – 5

4. Calculation of induced EMF ........................................................................... 6 – 20

5. Induced Electric Field due to a time Varying ................................................. 20 – 22

Magnetic field

6. Self Induction ............................................................................................... 23 – 24

7. Inductor ........................................................................................................ 24 – 26

8. L.R.Circuit ........................................................................................................ 27

9. Growth and Decay of current in L-R circuit .................................................... 28 – 32

10. Mutual Inductance ...................................................................................... 32 – 34

11. Series Combination of inductors ..................................................................... 35

12. Exercise - I ................................................................................................. 36 – 46

13. Exercise - II ................................................................................................ 47 – 50

14. Exercise - III ............................................................................................... 51 – 52

15. Exercise - IV .............................................................................................. 53 – 55

16. Exercise - V ............................................................................................... 56 – 59

17. Answer key ................................................................................................ 60 – 62

394,50 - Rajeev Gandhi Nagar Kota, Ph. No. : 93141-87482, 0744-2209671

Downloaded from www.iitjeephysics4u.com


IVRS No : 0744-2439051, 52, 53, www. motioniitjee.com, info@motioniitjee.com
Page # 2 ELECTROMAGNETIC INDUCTION

ELECTROMAGNETIC INDUCTION

In previous lesson we studied about magnetic field produced by a moving charge and the force on it when

placed in a magnetic field. In this lesson we will see how the current and emf are induced in a circuit when the

magnetic flux through the circuit changes with time.

In 1820, the connection between electricity and magnetism was demonstrated by Faraday and indepen-

dently by Joseph Henry. They showed that an electric current could be induced in a circuit by a changing

magnetic field.

The result of these experiments led to a very basic and important law of electromagnetism known as Faraday’s

law of induction.

In this lesson we discuss the ideas of flux and the basic law of producing induced emf. Then we shall discuss

inductor, which plays on important role in electrical circuits and its effect known as self induction and mutual

induction. Finally, we examine the characteristics of circuits containing inductors, resistors and capacitors

in various combinations.

IIT-JEE Syllabus : Faraday’s Law, Lenz’s Law; Motional Emf; Self and mutual inductance. RC and

LR circuits with d.c. sources.

394,50 - Rajeev Gandhi Nagar Kota, Ph. No. : 93141-87482, 0744-2209671

Downloaded from www.iitjeephysics4u.com


IVRS No : 0744-2439051, 52, 53, www. motioniitjee.com, info@motioniitjee.com
ELECTROMAGNETIC INDUCTION Page # 3

1. MAGNETIC FLUX
Consider a closed curve enclosing an area A (as shown in the figure). Let there be a uniform
 
magnetic field B in that region. The magnetic flux through the area A is given by

 
  B. A ...(i)
= BA cos 

where  is the angle which the vector B makes with the normal to the surface. If B is
 
perpendicular to A , then the flux through the closed area A is zero. SI unit of magnetic flux
is weber (Wb).

Notes :
Area vector is  to the surface
For open surface choose one direction as the area vector direction and stick to it for the
whole problem.
For closed surfaces outward normal is taken as area vector direction
Flux is basically count of number of lines crossing a surface

 B . ds  0 because magnetic field lines exists in closed loop.

Ex.1 Find flux passing through Area ?

B
37°

 
Sol. Since A is  to B
 
  B.A  0

2. FARADAY’S LAW OF ELECTROMAGNETIC INDUCTION


Whenever the flux of magnetic field through the area bounded by a closed conducting loop
changes, an emf is produced in the loop. The emf is given by
d
 ...(ii)
dt
 
where   B . dA the flux of magnetic field through the area.

The emf so produced drives an electric current through the loop. If the resistance of the loop
is R, then the current
 1 d
i  ...(iii)
R R dt

394,50 - Rajeev Gandhi Nagar Kota, Ph. No. : 93141-87482, 0744-2209671

Downloaded from www.iitjeephysics4u.com


IVRS No : 0744-2439051, 52, 53, www. motioniitjee.com, info@motioniitjee.com
Page # 4 ELECTROMAGNETIC INDUCTION

Ex.2 A coil is placed in a constant magnetic field. The magnetic field is parallel to the plane
of the coil as shown in figure. Find the emf induced in the coil.

Sol.  = 0 (always) since area is perpendicular to magnetic field.


 emf = 0

Ex.3 Find the emf induced in the coil shown in figure. The magnetic field is perpendicular to
the plane of the coil and is constant.
Area=A


B

Sol.  = BA (always) = const.


 emf = 0

Ex.4 Show that if the flux of magnetic induction through a coil changes from 1 to 2, then the
 2 – 1
charge q that flows through the circuit of total resistance R is given by q = ,
R
where R is the resistance of the coil.
d
Sol. Let  be the instantaneous flux. Then is the instantaneous rate of change of flux which is
dt
1  d 
equal to the magnitude of the instantaneous emf. so the current in the circuit |i| = R  dt  ,

dq
since the current is the rate of flow of charge, that is, i =
dt
tt
 1 d 
q =  idt or q    R . dt  dt
t 0

where  is the time during which change takes place. but at t = 0,  1, and at t = t,  = 2


1 2  2 – 1
d
 q=
R 1 =
R

3. LENZ’S LAW
The effect of the induced emf is such as to oppose the change in flux that produces it.
S S N N
V V V V
N N S S

i i
i i
(a) (c) (b) (d)
In figure (a & b) as the magnet approaches the loop, the flux through the loop increases. The
induced current sets up an induced magnetic field Bind whose flux opposes this change. The
direction of Bind is opposite to that of external field Bext due to the magnet.

394,50 - Rajeev Gandhi Nagar Kota, Ph. No. : 93141-87482, 0744-2209671

Downloaded from www.iitjeephysics4u.com


IVRS No : 0744-2439051, 52, 53, www. motioniitjee.com, info@motioniitjee.com
ELECTROMAGNETIC INDUCTION Page # 5

In figure (c & d) the flux through the loop decreases as the magnet moves away from the
loop, the flux due to the induced magnetic field tries to maintain the flux through the loop.
The direction of Bind is same as that of Bext due to magnet.
Lenz’s law is closely related to the law of conservation of energy and is actually a consequence
of this general law of nature. As the north pole of the magnet moves towards the loop an
induced current is produced. This opposes the motion of N-pole of the bar magnet. Thus, in
order to move the magnet toward the loop with a constant velocity an external force is to be
applied. The work done by this external force gets transformed into electric energy, which
induces current in the loop.
There is another alternative way to find the direction of current inside the loop which is
described below.
Figure shows a conducting loop placed near a long, straight wire carrying a current i as
shown. If the current increases continuously, then there will be an emf induced inside the
loop. Due to this induced emf, an electric current is induced. To determine the direction of
current inside the loop we put an arrow as shown. The right hand thumb rule shows that the
normal to the loop is going into the plane. Again the same rule shows that the magnetic field
at the site of the loop is also going into the plane of the diagram.

   
Thus B and d A are in the same direction. Therefore B . d A is positive if i increases, the

magnitude of flux  increases. Since magnetic flux  is positive and its magnitude increases,
d
is positive. Thus  is negative and hence the current is negative. Thus the current
dt
induced is opposite, to that of arrow.

Brain Teaser
Two identical coaxial circular loops carry equal currents circulating in the same direction.
What will happen to the current in each loop if the loops approach each other ?

Ex.5 Find the direction of induced current in the coil shown in figure. Magnetic field is
perpendicular to the plane of coil and it is increasing with time.


B

Sol. Inward flux is increasing with time. To oppose it outward magnetic field should be induced.
Hence current will flow in anticlockwise.
v
Ex.6 Figure shows a long current carrying wire and
two rectangular loops moving with velocity v.
Find the direction of current in each loop.
Sol. In loop (i) no emf will be induced because there is no flux (i)
change. constant i
ln loop (ii) emf will be induced because the coil is moving V
in a region of decreasing magnetic field inward in direction.
(ii)
Therefore to oppose the flux decrease in inward direction,
current will be induced such that its magnetic field will
be inwards. For this direction of current should be
clockwise.

394,50 - Rajeev Gandhi Nagar Kota, Ph. No. : 93141-87482, 0744-2209671

Downloaded from www.iitjeephysics4u.com


IVRS No : 0744-2439051, 52, 53, www. motioniitjee.com, info@motioniitjee.com
Page # 6 ELECTROMAGNETIC INDUCTION

4. CALCULATION OF INDUCED EMF

As we know that magnetic flux () linked with a closed conducting loop = BA cos  where B is
the strength of the magnetic field, A is the magnitude of the area vector and  is the angle
between magnetic field vector and area vector.
Hence flux will be affected by change in any of them, which is discussed in the next page.

4.1 By changing the magnetic field

Ex.7 Figure shows a coil placed in decreasing magnetic field applied perpendicular to the
plane of coil. The magnetic field is decreasing at a rate of 10 T/s. Find out current in
magnitude and direction

A=2m2

B R  5

Sol.  = B.A
dB
emf = A. = 2 × 10 = 20 v
dt
 i = 20/5 = 4amp. From lenz’s law direction of current will be anticlockwise.

Ex.8 The magnetic flux (2) in a closed circuit of resistance 20  varies with time (t) according
to the equation  = 7t2 – 4t where  is in weber and t is in seconds. The magnitude of
the induced current at t = 0.25 s is
(A) 25 mA (B) 0.025 mA (C) 47 mA (D) 175 mA
Sol.  = 7t2 – 4t
d
 Induced emf : | e |   14 t  4
dt
 Induced current :
| e | |14t  4| |14  0.25  4|
i   (at t = 0.25 s)
R 20 20
0.5
  2.5  102 A
20
 (A)

Ex.9 Consider a long infinite wire carrying a time varying current i = kt (k > 0). A circular
loop of radius a and resistance R is placed with its centre at a distance d from the wire
(a < < d). Find out the induced current in the loop?
Sol. Since current in the wire is continuously increasing therefore we conclude that magnetic field
due to this wire in the region is also increasing.
 0i
Magnetic field B due to wire  going into and perpendicular to the plane of the paper
2d
Flux through the circular loop,
i
  0  a2
2d i = kt a
 a2kt k>0 d
 0
2d
Induced e.m.f. in the loop
d  0 a2k
 
dt 2d
|  |  0a2k
Induced current in the loop i  
R 2dR
Direction of induced current in the loop is anticlokwise.

394,50 - Rajeev Gandhi Nagar Kota, Ph. No. : 93141-87482, 0744-2209671

Downloaded from www.iitjeephysics4u.com


IVRS No : 0744-2439051, 52, 53, www. motioniitjee.com, info@motioniitjee.com
ELECTROMAGNETIC INDUCTION Page # 7

Ex.10 Two concentric coplanar circular loops have diameters 20 cm and 2 m and resistance
of unit length of the wire = 10–4 /m. A time -dependent voltage V =(4 + 2.5 t) volts is
applied to the larger as shown. The current in the smaller loop is
V=(4+2.5t)

r1
r2

Sol. r1 = 1.0 m, r2 = 10–1 m


V=(4+2.5t)
Resistance of outer loop = 2 × 10–4 
Resistance of inner loop = 0.2 × 10–4 
V (4  2.5t) r1
Current in outer loop   A
R 2  104 r2

 2  4  1.25  4 
or i0      10      10  t  A
 
Magnetic field produced at the common centre (see figure)
 i
B 0 
2r1
4 [(2  1.25t)  104 ]
or B  107   2  103 (2  1.25t) T
2 
Hence, flux linked with the inner loop,
 = BA = 2 × 10–3 (2 + 1.25 t) × (0.1)2 = 2 × 10–5 (2 + 1.25t) Wb
Hence, the e.m.f. induced in smaller loop =
d
  2  10 5  125
.  –2.5   10 5 V
dt
The negative sign indicates that the induced e.m.f. (or current) is opposite to applied e.m.f.
(or current)
Hence, the current induced in the inner (smaller) loop is
| | 2.5  105 V
i   1.25 A
R (0.2  104 )

Ex.11 A rectangular wire frame of length 0.2 m, is located at a distance of 5 × 10–2 m from a
long straight wire carrying a current of 10 A as shown in the figure. The width of the
frame = 0.05 m. The wire is in the plane of the rectangle. Find the magnetic flux through
the rectangular circuit. If the current decays uniformly to 0 in 0.2 s, find the emf induced
in the circuit.
0.05m

10A
r1=0.05m
0.2m
x
dx
r2=0.1m
Sol. A current, i = 10 A is flowing in the long straight wire. Consider a small rectangular strip (in
the rectangular wire frame) of width dx at a distance x from the straight wire.
The magnetic flux at the location of the strip,
 0i
Bx =
2 x

394,50 - Rajeev Gandhi Nagar Kota, Ph. No. : 93141-87482, 0744-2209671

Downloaded from www.iitjeephysics4u.com


IVRS No : 0744-2439051, 52, 53, www. motioniitjee.com, info@motioniitjee.com
Page # 8 ELECTROMAGNETIC INDUCTION

The flux linked with the infinitesimally small rectangular strip


 0i
= Bx × Area of the strip = dx = l dx
2 x
where l is the length of the rectangular wire circuit
= 2 × 10–1 m
or dx = (0 il/2) (dx/x)
Hence, the total magnetic flux linked with the rectangular frame
0il
=  d x  [loge x]rr12
2

0il 0il  r2 
or   2 [loge r2  loge r1 ]  2 loge  r 
 1
Substituting values, we get
 = 2 × 10–7 × 10 × 2 × 10–1 × loge2
= 2.772 × 10–7 Wb

r 
 o l loge  2 
Induced e.m.f. |  | d   r1  di
dt 2 dt

10
 (2  107  2  101 loge 2)
0.2
= 1.386 ×10–6 V = 1.386 V

Ex.12 Figure shows a wire frame PQSTXYZ placed in a time varying magnetic field given as B
=  t, where  is a positive constant. Resistance per unit length of the wire is . Find the
current induced in the wire and draw its electrical equivalent diagram.
Y Z
b
b>a
B

T S
X

B a

P a Q

Sol. Induced emf in part PQST =  a2 (in anticlockwise  4b


direction, from Lenz’s Law) Similarly induced emf
i
in part TXYZ =  b2 (in anticlockwise direction,
2
from Lenz’s Law) b
Total resistance of the part PQST = a i i
Total resistance of the part PQST = l4b.
The equivalent circuit it is shown in the diagram. i  a2
writing KVL along the current flow i
b2 – a2 –  4ai –  4bi = 0  4a

i= (b – a)
4

Brain Teaser :
A copper ring is held horizontally and a bar magnet is dropped through the ring with its
length along the axis of the ring. Will the acceleration of the falling magnet be equal to,
greater than or lesser than the acceleration due to gravity ?

394,50 - Rajeev Gandhi Nagar Kota, Ph. No. : 93141-87482, 0744-2209671

Downloaded from www.iitjeephysics4u.com


IVRS No : 0744-2439051, 52, 53, www. motioniitjee.com, info@motioniitjee.com
ELECTROMAGNETIC INDUCTION Page # 9

4.2 BY CHANGING THE AREA


Solved Examples :
Ex.13 A space is divided by the line AD into two regions. Region I is field free and the region
II has a uniform magnetic field B directed into the paper. ACD is a semicircular
conducting loop of radius r with centre at O, the plane of the loop being in the plane of
t he paper . The loop is now m ade t o r ot at e w it h a const ant angular velocit y  about an
axis passing through O, and perpendicular to the plane of the paper in the clockwise
direction. The effective resistance of the loop is R.
(a) Obtain an expression for the magnitude of the induced current in the loop.
(b) Show the direction of the current when the loop is entering into the region II.
(c) Plot a graph between the induced emf and the time of rotation for two periods of
rotation.
Sol. (a) As in time t, the arc swept by the loop in the field, i.e., region II.
1 1
A= r(r)  r 2t
2 2
So the flux linked with the rotating loop at time t,
1
 = BA = Br 2 t [  t]
2
and hence the induced emf in the loop,
d 1 2
=– = – Br = constant.
dt 2
And as the resistance of the loop is R, the induced current in it,
 Br 2
i  
R 2R

Region I Region II + + + + +
Region I Region II
× × × × × × A + + + + +
 A
+ + + + +
× × × × × × 

+ + + + + 
T/2 t
C O × × × × × × O+ + + + + O
T 3T/2 2T
+ + + + +
× × × × × ×
–
D D+ + + + +
× × × × × × i
+ + + + + Fig.(c)
Fig (a) Fig (b)

(b) When the loop is entering the region II, i.e., the field figure (b), the inward flux linked
with it will increase, so in accordance with Lenz’s law an anticlockwise current will be
induced in it.
(c) Taking induced emf to the negative when flux linked with the loop is increasing and
positive when decreasing, the emf versus time graph will be, as shown in figure (c)

Ex.14 Two parallel, long, straight conductors lie on a smooth plane surface. Two other parallel
conductors rest on them at right angles so as to form a square of side a initially. A
uniform magnetic field B exists at right angles to the plane containing the conductors.
Now they start moving out with a constant velocity (v). (a) Will the induced emf be
time dependent? (b) Will the current be time dependent?
Sol. (a) Yes,  (instantaneous flux) = B (a + 2vt)2

d
 = = 4Bv(a + 2vt)
dt

394,50 - Rajeev Gandhi Nagar Kota, Ph. No. : 93141-87482, 0744-2209671

Downloaded from www.iitjeephysics4u.com


IVRS No : 0744-2439051, 52, 53, www. motioniitjee.com, info@motioniitjee.com
Page # 10 ELECTROMAGNETIC INDUCTION

(b) No,


(instantaneous current) i =
R
Now R = 4(a + 2vt)r where r = resistance per unit length

4Bv(a  2vt) Bv
 i= = (a constant)
4r(a  2vt) r
The current will be time independent

Ex.15 Find the direction of induced of current in the × × × ×


wire AB. When rotated anticlockwise ×A × × B×
through angle  if
× × × ×
it is placed initially as shown in the figure
× × × ×
Sol. Range of Angle Rotated Direction of induced current
×D × × ×
0 – 90 A to B C
× × × ×
90 – 180 A to B
180 – 270 B to A
270 – 360 B to A

4.3 Motional Emf


We can find emf induced in a moving rod by considering the number of lines cut by it per sec
assuming there are ‘B’ lines per unit area. Thus when a rod of length l moves with velocity v
in a magnetic field B, as shown, it will sweep area per unit time equal to lv and hence it will
cut B l v lines per unit time.
t t+dt

l V B

Hence emf induced between the ends of the rod = Bvl

d
Also emf = . Here  denotes flux passing through the area, swept by the rod. The rod
dt
d
sweeps an area equal to l vdt in time interval dt. Flux through this area = Bl vdt. Thus =
dt
Bl vdt
= Bvl
dt
If the rod is moving as shown in the following figure, it will sweep area per unit time = v l sin

t t+dt


l B

and hence it will cut B v l sin  lines per unit time. Thus emf = Bvlsin.

394,50 - Rajeev Gandhi Nagar Kota, Ph. No. : 93141-87482, 0744-2209671

Downloaded from www.iitjeephysics4u.com


IVRS No : 0744-2439051, 52, 53, www. motioniitjee.com, info@motioniitjee.com
ELECTROMAGNETIC INDUCTION Page # 11

4.3.1 Mechanism of The induced EMF a cross the ends of a moving rod :
Figure shows a conducting rod of length l moving with a constant velocity v in a uniform
magnetic field. The length of the rod is perpendicular to magnetic field, and velocity is
perpendicular to both the magnetic field and the length of the rod. An electron inside the
 
conductor experiences a magnetic force FB   e ( v  B) directed downward along the rod. As a
result electrons migrate towards the lower end and leave unbalanced positive charges at the
top. This redistribution of charges sets up an electric field E directed downward. This electric
field exerts a force on free electrons in the upward direction. As redistribution continues
electric field grows in magnitude until a situation, when
Q +
× × FB = e

× ×
× ×
× × v
× ×
× ×
× ×
FE = eE
× × P –
  
| q v  B |  | qE|
After this, there is no resultant force on the free electrons and the potential difference
across the conductor is
    
 d    E.d l   (v  B).d l ...(4)
Thus it is the magnetic force on the moving free electrons that maintains
the potential difference. So e.m.f. developed across the ends of the P
rod moving perpendicular to magnetic field with velocity perpendicular
Blv
to the rod,
  vB l ...(5)
r
As this emf is produced due to the motion of the conductor, it is called
motional emf.
In the problems related to motional e.m.f. we can replace the rod by a Q
battery of e.m.f. vB l.
The moving rod can be represented (or equivalent) as electrical circuit
as shown in figure.

Ex.16 Find the value of emf induced in the rod for the following cases. The figures are self
explanatory.
V

(a) Ans. emf = 0


B

(b) B Ans. emf = 0

(c) Ans. emf = 0

Figure shows a closed coil ABCA moving in a uniform magnetic field B with a velocity v. The
flux passing through the coil is a constant and therefore the induced emf is zero.

394,50 - Rajeev Gandhi Nagar Kota, Ph. No. : 93141-87482, 0744-2209671

Downloaded from www.iitjeephysics4u.com


IVRS No : 0744-2439051, 52, 53, www. motioniitjee.com, info@motioniitjee.com
Page # 12 ELECTROMAGNETIC INDUCTION

A A
B
L C
C V E VBL

B B
Now consider rod AB, which is a part of the coil. Emf induced in the rod = B L v. Now suppose
the emf induced in part ACB is E, as shown in figure.
Since the emf in the coil is zero, Emf (in ACB) + Emf (in BA) = 0
or – E + vBL = 0
or E = vBL
Thus emf induced in any path joining A and B is same, provided the magnetic field is uniform.
Also the equivalent emf between A and B is BLv (here the two emf’s are in parallel)

Ex.17 Figure shows an irregular shaped with AB moving with velocity v, as shown. Find the
emf induced in the wire.
v A

l
B
B A v A

 B l
l
Sol. The same emf will be induced in the straight imaginary wire B B
joining A and B, which is Bvl sin  B

Ex.18 A 0.4 meter long straight conductor moves in a magnetic field of magnetic induction
0.9 Wb/m2 with a velocity of 7 m/sec. Calculate the emf induced in the conductor
under the condition when it is maximum.

Sol. If a rod of length l is moved with velocity v and at angle  to the length of the rod in a field

B which is perpendicular to the plane of the motion, the flux linked with the area generated
by the motion of rod in time t,
v
× × × × ×

× × × × ×

× × ×  × ×

× × l× × ×

d
 = Bl(v sin)t so, || = = Bv l sin 
dt
This will be maximum when sin  = max = 1, i.e., the rod is moving perpendicular to its length
and then ()max = Bvl
Emax = 0.9 × 7 × 0.4 = 2.52 V

394,50 - Rajeev Gandhi Nagar Kota, Ph. No. : 93141-87482, 0744-2209671

Downloaded from www.iitjeephysics4u.com


IVRS No : 0744-2439051, 52, 53, www. motioniitjee.com, info@motioniitjee.com
ELECTROMAGNETIC INDUCTION Page # 13

Ex.19 A square metal wire loop of side 10 cm and resistance 1 ohm is moved with a constant
velocity v0 in a uniform magnetic field of induction B = 2 Wb/m2 as shown in figure.
The magnetic field lines are perpendicular to the plane of the loop. The loop is connected
to a network of resistance each of value 3 ohm. The resistances of the lead wires OS
and PQ are negligible. What should be the speed of the loop so as to have a steady
current of 1 milliampere in the loop ? Find the direction of current in the loop ?
v0

× × × × × × × × Q
 × × × × × × × × 3 3
B × × × × × × × × P 3
× × × × × × × × A C
× × × × × × × × O
× × × × × × × × 3 3
× × × × × × × ×
× × × × × × × × S

Sol. As the network AQCS is a balanced Wheatstone bridge, no current will flow through AC and
hence the effective resistance of the network between QS,
66
RQS = = 3 ohm
66
and as the resistance of the square metal wire loop is 1 ohm, the total resistance of the
circuit,
R = 3 + 1 = 4 ohm
Now if the loop moves with speed v0, the emf induced in the loop,
 = Bv0l
 Bv0l
So the current in the circuit, i = =
R R
Substituting the given data,
In accordance with Lenz’s law, the induced current in the loop will be in clockwise direction.
Ex.20 A rod of length l is kept parallel to a long wire carrying constant current i. It is moving
away from the wire with a velocity v. Find the emf induced in the wire when its distance
from the long wire is x.
 0il v
Sol. E=BlV=
2 x
OR
Emf is equal to the rate with which magnetic field lines are cut. In dt time the area swept by
the rod is l v dt. The magnetic field lines cut in dt time
t
0il vdt B
= B l vdt = l v
2x i
 The rate with which magnetic field lines are cut Const x
 il v
= 0
2 x
Ex.21 A rectangular loop, as shown in the figure, moves away from an infinitely long wire
carrying a current i. Find the emf induced in the rectangular loop.
E = B1 L V – B2 L v

L v
i B1LV B2LV
x b

0i  0i  0iLbv
= Lv – Lv =
2x 2(x  b) 2x( x  b)

394,50 - Rajeev Gandhi Nagar Kota, Ph. No. : 93141-87482, 0744-2209671

Downloaded from www.iitjeephysics4u.com


IVRS No : 0744-2439051, 52, 53, www. motioniitjee.com, info@motioniitjee.com
Page # 14 ELECTROMAGNETIC INDUCTION

Aliter : Consider a small segment of width dy at a distance y from the wire. Let flux through the
segment be d.

0i dy
 d = L dy
2y
y L
v
x b constant
 0iL dy  0iL current i
 =
2 
x
y
=
2
(ln(x  b) – ln x)
x

d  0iL  1 dx 1 dx   0iL  (–b)  – 0ibLv


Now dt  2  x  b dt – x dt  = v =
  2  x(x  b)  2x(x  b)

 0ibLv
 induced emf =
2x(x  b)

Ex.22 A rod of length l is placed perpendicular to a long wire carrying current i. The rod is
moved parallel to the wire with a velocity v. Find the emf induced in the rod, if its
nearest end is at a distance ‘a’ from the wire.
Sol. Consider a segment of rod of length dx, at a
distance x from the wire. Emf induced in the
segment v
B
0i + –
d = dx.v
2x a
l
a l
 0ivdx 0iv  l  a 
 =   ln 
a
2x 2  a 

Ex.23 Two parallel wires AL and BM placed at a distance l are connected by a resistor R and
placed in a magnetic field B which is perpendicular to the plane containing the wires.
Another wire CD now connects the two wires perpendicularly and made to slide with
velocity v. Calculate the work done per second needed to slide the wire CD. Neglect
the resistance of all the wires.
Sol. When a rod of length l moves in a magnetic field with
C
velocity v as shown in figure, an emf  = Bvl will be A L
induced in it. Due to this induced emf, a current × × × × × × × × × ×
 Bvl
i= = will flow in the circuit as shown in figure. × × × × × × × × × ×
R
R R l FM v
Due to this induced current, the wire will experience a × × × × × × × × × ×
magnetic force × × × × × × × × × ×
B2 l 2 v B M
FM = Bi l = D
R
which will oppose its motion, So to maintain the motion of the wire CD, a force F = FM must be
applied in the direction of motion.
The work done per second, i.e., power needed to slide the wire is given by
dW B2 v 2 l 2
P= = Fv = FM v =
dt R
Note :
• The power delivered by the external agent is converted into joule heating in a by using the
circuit(as shwon above). It means magnetic field helps in converting the mechanical energy
into joule heating.

394,50 - Rajeev Gandhi Nagar Kota, Ph. No. : 93141-87482, 0744-2209671

Downloaded from www.iitjeephysics4u.com


IVRS No : 0744-2439051, 52, 53, www. motioniitjee.com, info@motioniitjee.com
ELECTROMAGNETIC INDUCTION Page # 15

Ex.24 A rod of mass m and resistance r is placed on fixed, resistanceless, smooth conducting
rails (closed by a resistance R) and it is projected with an initial velocity u Find its
velocity as a function of time.
t=0
B
l u R
m,r

i
Sol. Let at an instant the velocity of the rod be v. The emf induced
in the rod will be vBl. The electrically equivalent circuit is shown
r R
in the following diagram.
Bl v
 Current in the circuit i = Blv
R r
At time t
Magnetic force acting on the rod is F = i l B, opposite to the motion of the rod.
dv
ilB=–m ...(1)
dt
Bl v
i= ...(2)
R r
Now solving these two equation V=ue
–kt

B2 l 2 v dv B2 l 2 dv u
 –m.  – .dt 
R r dt (R  r) m v
v
dV
let  – K. dt =
V
v
dv
t
t
   –K.dt
u
v 0

 v
ln  u   –Kt  V = ue–Kt

Ex.25 In the above question if a constant force F is applied on the rod. Find the velocity of the
rod as a function of time assuming it started with zero initial velocity.
dv
Sol. m =F–ilB ..(1)
dt
Bl v
i= ...(2)
R r
dv B2 l 2 v
m =F–
dt R r
v t
B2 l 2 dV dt Vmax
let K =    m
R r 0
F – Kv 0
V
v
1 t
 – ln(F – KV)  t
K 0
m

 F – kV  Kt
ln  –
 F  m
F – KV = F e–kt/m
F
V= (1 – e–kt/m)
K

394,50 - Rajeev Gandhi Nagar Kota, Ph. No. : 93141-87482, 0744-2209671

Downloaded from www.iitjeephysics4u.com


IVRS No : 0744-2439051, 52, 53, www. motioniitjee.com, info@motioniitjee.com
Page # 16 ELECTROMAGNETIC INDUCTION

Ex.26 A square wire of length l, mass m and resistance R slides without friction down the
parallel conducting wires of negligible resistance as shown in figure.
The rails are connected to each other at the bottom by a resistanceless rail parallel to
the wire so that the wire and rails form a closed rectangular loop. The plane of the rails
makes an angle  with horizontal and a uniform vertical field of magnetic induction B
exists throughout the region. Show that the wire acquires a steady state velocity of
mgRsinθ
magnitude v = 2 2
B l cos 2 θ
B

Sol. Force down the plane = mg sin 


At any instant if the velocity is v the induced e.m.f
= l B cos × v
Bl v cos 
Current in the loop ×
R
Force on the conductor in the horizontal direction
Bl v cos 
= Bl ×
R
The component of force parallel to the incline
B2 l 2 v cos 
=  cos 
R
B2 l 2 cos2 
If v is constant,  v = mg sin 
R
mRgsinθ
 v=
B 2 l 2 cos 2 θ

Ex.27 A rod PQ of mass m and resistance r is moving on two fixed, resistanceless, smooth
conducting rails (closed on both sides by resistances R1 and R2). Find the current in the
rod at the instant its velocity is v.
P

B r
R1 l V R2

Q
Sol. The equivalant circuit of above figure

Biv
R1 r R2

Bl v
i= R1R 2
r
R1  R 2

394,50 - Rajeev Gandhi Nagar Kota, Ph. No. : 93141-87482, 0744-2209671

Downloaded from www.iitjeephysics4u.com


IVRS No : 0744-2439051, 52, 53, www. motioniitjee.com, info@motioniitjee.com
ELECTROMAGNETIC INDUCTION Page # 17

Ex.28 In the above question if one resistance is replaced


by a capacitor of capacitance C as shown. Find the
velocity of the moving rod at time t if the initial
velocity of the rod is v and a constant force F is R F C
applied on the rod. Neglect the resistance B
of the rod.
Sol. At any time t, let the velocity of the rod be v.
Applying Newtons law :
F – i lB = ma ...(1)
q
Also B l v = i1 R = dq
c
i1 dt
Applying KCL,
i
dq Bl v d Bl v F q
i = i1 +   (Bl vC) or i=  Bl Ca R V
dt R dt R C
ilB –q
Putting the value of i in eq. (1), B

B2 l 2 v dv
F– = (m + B2l2C)a = (m + B2l2C)
R dt
dv
(m + B2l2C)  dt
B2l 2 v
F–
R
Integrating both sides, and solving we get

 tB2 l 2 
FR R(m+CB2 l 2 )
v = B2 l 2  1 – e 
 

4.4 Induced e.m.f due to rotation

4.4.1 Rotation of the rod


Consider a conducting rod of length l rotating in a uniform magnetic field.


B B d  rBdr
dr
dr v  r
l emf induced in a
r small segment small segment
just like a rod

Emf induced in a small segment of


length dr, of the rod = v B dr = r  B dr
 emf induced in the rod
 1 2 r
=B  rdr  2 Bl
0

Bl 2 / 2
equivalent of this rod is as following

flux through the area swept by


d the rod in time dt
or  = =
dt
dt
1
B l 2 dt
2 1 2
= = Bl
dt 2

394,50 - Rajeev Gandhi Nagar Kota, Ph. No. : 93141-87482, 0744-2209671

Downloaded from www.iitjeephysics4u.com


IVRS No : 0744-2439051, 52, 53, www. motioniitjee.com, info@motioniitjee.com
Page # 18 ELECTROMAGNETIC INDUCTION

Ex.29 Find out the potential difference between A & B :

× × × × × × ×
× × × × × × ×
× × × × × × ×
A B
× × × × × × ×
l
× × × × × × ×
× × × × × × ×
Sol.

× × × × × × ×
× × × × × × ×
× × × × dx× × ×
A B
× × × x × × × ×
l
× × × × × × ×
× × × × × × ×

 dE   Bxdx
0

Bl 2
VA – VB 
2

Ex.30 A rod PQ of length 2l is rotating about one end P in a uniform magnetic field B which is
perpendicular to the plane of rotation of the rod. Point M is the mid point of the rod.
Find the induced emf between M & Q if that between P & Q = 100V.
 B = Uniform
Q
P M

2l
Bwl 2
Sol. EMQ + EPM = EPQ corner  = 100
2
2
 l
B  
 2 Bl 2
EMQ + =
2 2
3 3
EMQ = Bl 2 =  100 V = 75 V
8 4

Ex.31 A rod of length L and resistance r rotates about one end as shown in figure. Its other
end touches a conducting ring a of negligible resistance. A resistance R is connected
between centre and periphery. Draw the electrical equivalence and find the current in
the resistance R. There is a uniform magnetic field B directed as shown.
B

394,50 - Rajeev Gandhi Nagar Kota, Ph. No. : 93141-87482, 0744-2209671

Downloaded from www.iitjeephysics4u.com


IVRS No : 0744-2439051, 52, 53, www. motioniitjee.com, info@motioniitjee.com
ELECTROMAGNETIC INDUCTION Page # 19

O r
A r
E 1
r Bl 2 i
O A  R 2
1 D R
Sol. Bl 2
R 2
D C E
C
1
Bl 2
current i = 2
R r
E 
Ex.32 Solve the above question if the length of rod B
is 2L and resistance 2r and it is rotating about
O
its centre. D A
L
Both ends of the rod now touch the conducting ring.
R

E C
r r/2
r r 
A
1
R
C,E,A O  BL2
r

 2
Sol. C  E  i=
r r
R R
E R 2 2
R

Ex.33 A rod of length l is rotating with an angular speed  about its one end which is at a
distance ‘a’ from an infinitely long wire carrying current i. Find the emf induced in the
rod at the instant as shown in the figure.

l
i

Consider a small segment of rod of length dx, at a


distance x from one end of the rod.
Emf induced in the segment a x

 0i i
dE = (x)dx
2(x  a) dx
l x
 0i  i  l – a.ln  l  a  
E=  (x)dx = 0   a  
0
2(x  a) 2  
4.5 By Changing The Angle
Let us consider the case when the magnitude of the magnetic field strength and the area of
the coil remains constant. When the coil is rotated relative to the direction of the field, an
induced current is produced which lasts as long as the coil is rotating.
We have,  = BA cos  [where B is the magnetic field strength, A is the magnitude of the area
vector &  is the angle between them] If the angular velocity with which the coil is rotating is
, then  = t B
Induced e.m.f. in the coil A
d
  BA sin t
dt
Induced current in the coil
|  | BA
i  sin t
R R
394,50 - Rajeev Gandhi Nagar Kota, Ph. No. : 93141-87482, 0744-2209671

Downloaded from www.iitjeephysics4u.com


IVRS No : 0744-2439051, 52, 53, www. motioniitjee.com, info@motioniitjee.com
Page # 20 ELECTROMAGNETIC INDUCTION

Ex.34 A ring rotates with angular velocity  about an axis in the   const
plane of the ring and which passes through the center of
the ring. A constant magnetic field B exists perpendicular
to the plane of the ring. Find the emf induced in the ring as
B
a function of time. t=0
Sol. At any time t,  = BA cos  = BA cos t
Now induced emf in the loop

–d
e= = BA  sin t
dt
If there are N turns
emf = BAN sin t
BA N is the amplitude of the emf e = em sin t

e em
i= = sin t = im sin t
R R

em
im =
R
The rotating coil thus produces a sinusoidally varying current or alternating current. This is
the principle which is always used in generator.

5. INDUCED ELECTRIC FIELD DUE TO A TIME VARYING MAGNETIC FIELD



Consider a conducting loop placed at rest in a magnetic field B . Suppose, the field is constant
till t = 0 and then changes with time. An induced current starts in the loop at t = 0.
The free electrons were at rest till t = 0 (we are not interested in the random motion of the
electrons.) The magnetic field cannot exert force on electrons at rest. Thus, the magnetic
force cannot start the induced current. The electrons may be forced to move only by an
electric field. So we conclude that an electric field appears at time t = 0.
This electric field is produced by the changing magnetic field and not by charged
particles. The electric field produced by the changing magnetic field is nonelectrostatic
and nonconservative in nature. We cannot define a potential corresponding to this
field. We call it induced electric field. The lines of induced electric field are closed
curves. There are no starting and terminating points of the field lines.
 
If E be the induced electric field, the force on the charge q placed in the field of q E . The
 
work done per unit charge as the charge moves through d l is E. d l .The emf developed in
the loop is, therefore,
 
=  E .d l
Using Faraday’s law of induction,

d
=–
dt
  d
or,  E .d l =–
dt
...(6)
The presence of a conducting loop is not necessary to have an induced electric field. As long

as B keeps changing, the induced electric field is present. If a loop is there, the free
electrons start drifting and consequently an induced current results.

394,50 - Rajeev Gandhi Nagar Kota, Ph. No. : 93141-87482, 0744-2209671

Downloaded from www.iitjeephysics4u.com


IVRS No : 0744-2439051, 52, 53, www. motioniitjee.com, info@motioniitjee.com
ELECTROMAGNETIC INDUCTION Page # 21

Ex.35 What will be the electric field at a distance r B=B0t


from axis of changing cylindrical magentic field
×
B, which is parallel to the axis of cylinder ? × ×
× ×
Sol. (i) When r < R × × × ×
× R× ×
let at a distance r electric field is E × ×
  d ×E
  E.dl  – E× × × B
dt
r× ×
× ×
××
d[B.(r 2 )] ×
E. 2r = – dt
× ×R
×

r dB B0r
E =
2 dt 2
(ii) When r > R
E E
  –d ×
 E.d   × R
dt ×
×× ×
r
d(BR 2 )
E.2 r =
dt
E
R2 dB B R2
 E . = 0
2r dt 2r

Ex.36 Find the e.m.f induced in the rod as shown in the figure.
B=B0t

× ×
× × ×
×
Circular field region
× × R (Radius = R)
× d ×
×
A ×  ×
× B

d sec 

 d

90 – 
Sol.
dx x 

394,50 - Rajeev Gandhi Nagar Kota, Ph. No. : 93141-87482, 0744-2209671

Downloaded from www.iitjeephysics4u.com


IVRS No : 0744-2439051, 52, 53, www. motioniitjee.com, info@motioniitjee.com
Page # 22 ELECTROMAGNETIC INDUCTION

r dB d sec 
E  B0
2 dt 2

d sec B0 dx cos 


dE = E.dx cos  
2

E
B0 d  / 2 B0  d
  dE 
0
2 –/ 2
dx  E
2
Alternate
 
Induced emf in OA & OB is zero (because B & dl are perpendicular)
Total induced emf in OAB is in AB
1
Area = ld
2
O
B0 l dt
=
2
A l B
d B0 l d

dt 2

Ex.37 A thin, nonconducting ring of mass m, carrying a charge q, can rotate freely about its
axis. At the instant t = 0 the ring was at rest and no magnetic field was present. Then
suddenly a magnetic field B was set perpendicular to the plane. Find the angular velocity
acquired by the ring.
Sol. Due to the sudden change of flux, an electric field is set up and the ring experiences an
impulsive torque and suddenly acquires an angular velocity.

d d 
(induced emf) =   B .dA
dt dt 
 
Also   E. d l where E is the induced electric field.

  d   d
  E.d l  B .dA  E.2r   (Br 2 )
dt  dt

r dB
 E
2 dt

Force experienced by the ring = q |E|

qr 2 dB
Torque experienced by the ring   (qE)r 
2 dt

 Angular impulse experienced by the ring

qr 2 dB B
   dt  dt  qr2
2  dt 2
Also angular impulse acquired = l where l is moment of inertia of the ring about its axis = mr2
 mr2  = qr2 B/2
 Angular velocity acquired by the ring  = qB/2m

394,50 - Rajeev Gandhi Nagar Kota, Ph. No. : 93141-87482, 0744-2209671

Downloaded from www.iitjeephysics4u.com


IVRS No : 0744-2439051, 52, 53, www. motioniitjee.com, info@motioniitjee.com
ELECTROMAGNETIC INDUCTION Page # 23

6. SELF INDUCTION

Self induction is induction of emf in a coil due to its own current change. Total flux N passing
through a coil due to its own current is proportional to the current and is given as N  = L i
where L is called coefficient of self induction or inductance. The inductance L is purely a
geometrical property i.e., we can tell the inductance value even if a coil is not connected in
a circuit. Inductance depends on the shape and size of the loop and the number of turns it
has.
If current in the coil changes by I in a time interval t, the average emf induced in the coil
is given as
(N) (LI) L l
= – = – –
t t t
The instantaneous emf is given as
d(N) d(LI) Ldl
=– =– –
dt dt dt
S.I unit of inductance is wb/amp or Henry (H)
L - self inductance is +ve quantity.
L depends on : (1) Geometry of loop
(2) Medium in which it is kept. L does not depend upon current. L is a scalar
quantity.
Brain Teaser
If a circuit has large self-inductance, what inference can you draw about the circuit.

6.1 Self Inductance of solenoid


l
r
l >> r
A
I
Let the volume of the solenoid be V, the number of turns per unit length be n. Let a current I
be flowing in the solenoid. Magnetic field in the solenoid is given as B = 0 nl. The magnetic
flux through one turn of solenoid  = 0 n l A.
The total magnetic flux through the solenoid = N 
= N 0 n l A
= 0 n2 l A l
 L = 0 n l A = 0 n2 V
2

 = 0 n i r2 (n l)

L= = 0 n2 r2 l
i
Inductance per unit volume = 0 n2

Ex.38 The current in a coil of self-inductance L = 2H is increasing according to the law i = 2 sin
t2. Find the amount of energy spent during the period when the current changes from
0 to 2 ampere.
Sol. Let the current be 2 amp at t = 

Then 2 = 2 sin 2   =
2
di
When the instantaneous current is i, the self induced emf is L . If the amount of charge
dt
that is displaced in time dt is dq, then the elementary work done

394,50 - Rajeev Gandhi Nagar Kota, Ph. No. : 93141-87482, 0744-2209671

Downloaded from www.iitjeephysics4u.com


IVRS No : 0744-2439051, 52, 53, www. motioniitjee.com, info@motioniitjee.com
Page # 24 ELECTROMAGNETIC INDUCTION

 di  di
= L .  dt  dq  L . dt idt  Lidi

 

W=  Lidi =  L(2 sin t


2
)d(2 sin t2 )
0 0

 
2 2 2
W =  8 L sin t cos t (tdt) = 4L  sin2t (tdt)
0 0

Let  = 2t2
Differentiating d = 4t dt
sin d
 W = 4L 
4
= L (– cos) = – L cos 2t2
 /2
W = –L cos 2t2 
0

= 2 L = 2 × 2 = 4 joule

7. INDUCTOR

It is represent by

L
electrical equivalence of loop
R
L

i i
If current i through the inductor is increasing the induced emf will oppose the increase in
current and hence will be opposite to the current. If current i through the inductor is decreasing
the induced emf will oppose the decrease in current and hence will be in the direction of the
current.
i
(increasing) i (decreasing) L
L

i i

induced emf
Induced emf
Over all result
i
A + – B
di
L
dt
di
VA – L = VB
dt

Note
If there is a resistance in the inductor (resistance of the coil of inductor) then :
A B A B
L, R L R

394,50 - Rajeev Gandhi Nagar Kota, Ph. No. : 93141-87482, 0744-2209671

Downloaded from www.iitjeephysics4u.com IVRS No : 0744-2439051, 52, 53, www. motioniitjee.com, info@motioniitjee.com
ELECTROMAGNETIC INDUCTION Page # 25

Ex.39 A B is a part of circuit. Find the potential difference vA – vB if

i 2

A B
1H 5 volt
(i) current i = 2A and is constant
(ii) current i = 2A and is increasing at the rate of 1 amp/sec.
(iii) current i = 2A and is decreasing at the rate 1 amp/sec.
2i
+ –
Sol.
A + – + – B
1H 5 volt
di di
L = 1
dt dt
writing KVL from A to B
di
VA – 1 – 5 – 2 i = VB
dt
di
(i) Put i = 2, =0
dt
VA – 5 – 4 = VB  VA – VB = 9 volt
di
(ii) Put i = 2, =1;
dt
VA – 1 – 5 – 4 = VB or VA – VB = 10 V0
di
(iii) Put i = 2, =–1
dt
VA + 1 – 5 – 2 × 2 = VB
VA = 8 volt
Ex.40 Find current i, i1 and i2 in the following circuit.
i R
i1
i2

V R
R
i R
Sol. at t = 0 i2
i1
V
i = i2 = and i1 = 0
2R V R
at t=
i R

i1 i2

V R R

i V
 i1 = i2 = =
2 2R

394,50 - Rajeev Gandhi Nagar Kota, Ph. No. : 93141-87482, 0744-2209671

Downloaded from www.iitjeephysics4u.com


IVRS No : 0744-2439051, 52, 53, www. motioniitjee.com, info@motioniitjee.com
Page # 26 ELECTROMAGNETIC INDUCTION

7.1 Energy stored in an inductor :


If current in an inductor at an instant is i and is increasing at the rate di/dt, the induced emf
will oppose the current. Its behaviour is shown in the figure.
working as a load

i, increasing i
 di / dt L di/dt

di
Power consumed by the inductor = i L
dt
di
Energy consumed in dt time = i L dt
dt
i
1 2
 total energy consumed as the current increases from 0 to I =  ILdi  2 Li
0

1 2 1
= L i  U = L i2
2 2

Note :
This energy is stored in the magnetic field with energy density
dU B2 B2
 
dV 2 2 0 r

B2
Total energy U =  2 dV
0 r

Ex.41 Find out the energy per unit length ratio inside
the solid long wire having current density J. R

Sol. Take a ring of radius r and thickness dr as an element inside the wire
dE B2
= 2
dv 0
r
 jr
using B= 0
2 dr

dE 2 J2r2  0 j2r2 2 4
E  0 j R
 0
dv 4(2 0 )
  dE   8
2rdr 


16

Ex.42 A circuit contains an ideal cell and an inductor with a


switch. Initially the switch is open. It is closed at t =
0. Find the current t=0
as a function of time.  L
i i
di
Sol. = L   dt   Ldi
dt 0 0

t
t = Li  i =
L

394,50 - Rajeev Gandhi Nagar Kota, Ph. No. : 93141-87482, 0744-2209671

Downloaded from www.iitjeephysics4u.com


IVRS No : 0744-2439051, 52, 53, www. motioniitjee.com, info@motioniitjee.com
ELECTROMAGNETIC INDUCTION Page # 27

8. L.R. CIRCUIT
As the switch S is closed in given figure, current in
L R
v
circuit wants to rise upto in no time but inductor
R
 di V
opposes it   
dt L
hence at time t = 0 inductor will behave as an open circuit S
v
at t = 0

di
As the time passes, i in the circuit rises and decreases. At any instant t.
dt
Ldi
 iR  V
dt
v
current reaches the value at time t =  or we can say, inductor will behave as a simple
R
wire.
at t = 

v/R R

i2 R
v
Ex.43 Find value of current i, i1 and i2 in given figure at i1
(a) time t = 0
L R
(b) time t =  i

v
R
Sol. (a) At time t = 0 inductor behaves as open circuit
i = v/R i2
i1 = 0 i1
i2= i = v/R
i
(b) At time t = . Inductor will behaves as simple wire
v
v 2v
i= 
(R / 2) R
i2 R

i1
v R
i1 = i2 = i
R

394,50 - Rajeev Gandhi Nagar Kota, Ph. No. : 93141-87482, 0744-2209671

Downloaded from www.iitjeephysics4u.com


IVRS No : 0744-2439051, 52, 53, www. motioniitjee.com, info@motioniitjee.com
Page # 28 ELECTROMAGNETIC INDUCTION

9. GROWTH AND DECAY OF CURRENT IN L-R CIRCUIT

9.1 Growth of Current


Consider a circuit containing a resistance R, an inductance L, a two way key and a battery of
e.m.f E connected in series as shown in figure. When the switch S is connected to a, the
current in the circuit grows from zero value. The inductor opposes the growth of the current.
This is due to the fact that when the current grows through inductor, a back e.m.f. is
developed which opposes the growth of current in the circuit. So the rate of growth of
current is reduced. During the growth of current in the circuit, let i be the current in the
circuit at any instant t. Using Kirchhoff’s voltage law in the circuit, we obtain

R L b
S
E a

i
di di
E–L = R i or E – Ri = L
dt dt
di dt
or 
E – Ri L
Multiplying by – R on both the sides, we get
–R di –Rdt

E – Ri L
Integrating the above equation, we have
R
loge(E – Ri) = – t +A ...(1)
L
where A is integration constant. The value of this constant can be obtained by applying the
condition that current i is zero just at start i.e., at t = 0. Hence
loge E = 0 + A
or A = logeE ...(2)
Substituting the value of A from equation (2) in equation (1), we get
R
loge(E – Ri) = – t + loge E
L
 E – Ri  R
or loge  E  = – t
L
 E – Ri   R 
or  E  = exp  – L t

Ri  R 
or 1– = exp  – t
E L

Ri   R 
or  1 – exp  – t 
E   L 

E  R 
 i = R 1 – exp  – L t 
 
The maximum current in the circuit i0 = E/R. So
  R 
i = i0 1 – exp  – L t  ...(3)
 
Equation (3) gives the current in the circuit at any instant t. It is obvious from equation (3)
that i = i0, when

394,50 - Rajeev Gandhi Nagar Kota, Ph. No. : 93141-87482, 0744-2209671

Downloaded from www.iitjeephysics4u.com


IVRS No : 0744-2439051, 52, 53, www. motioniitjee.com, info@motioniitjee.com
ELECTROMAGNETIC INDUCTION Page # 29

 R 
exp  – t = 0 i.e., at t = 
 L 
i0

i
Growth of current

O t
Hence the current never attains the value i0 but it approaches it asymptotically. A graph
between current and time is shown in figure.
• We observe the following points
(i) When t = (L/R) then
  R L   1
i = i0 1 – exp  – L  R   = i0 {1 – exp(–1)} = i0  1 –  = 0.63 i0
  e
Thus after an interval of (L/R) second, the current reaches to a value which is 63% of the
maximum current. The value of (L/R) is known as time constant of the circuit and is represented
by . Thus the time constant of a circuit may be defined as the time in which the current rises
from zero to 63% of its final value. In terms of ,
t
 
i  i0  1  e  
 
(ii) The rate of growth of current (di/dt) is given by

di d  i 1 – exp  – R  
t  
= 0  L
dt dt     

di R  R 
 dt  i0  L  exp  – L t ...(4)

 R  i –i
From equation (3), exp  – L t = 0
 i0

di  R   i0 – i  R
 dt  i0  L   i  = (i – i) ...(5)
 0  L 0
This shows that the rate of growth of the current decreases as i tends to i0. For any other
value of current, it depends upon the value of R/L. Thus greater is the value of time constant,
smaller will be the rate of growth of current.

Note :

Final current in the circuit =
, which is independent of L.
R
After one time constant, current in the circuit=63% of the final current (verify yourself)
More time constant in the circuit implies slower rate of change of current.
If there is any change in the circuit containing inductor then there is no instantaneous effect
on the flux of inductor.
L1i1 = L2i2

394,50 - Rajeev Gandhi Nagar Kota, Ph. No. : 93141-87482, 0744-2209671

Downloaded from www.iitjeephysics4u.com


IVRS No : 0744-2439051, 52, 53, www. motioniitjee.com, info@motioniitjee.com
Page # 30 ELECTROMAGNETIC INDUCTION

Ex.44 At t = 0 switch is closed (shown in figure) after a long time suddenly the inductance of
L
the inductor is made  times lesser ( ) then its initial value, find out instant current

just after the operation.
R L

S 
Sol. Using above result (note 4)

L1i1 = L2i2  i2 =
R
Ex.45 Which of the two curves shown has less time constant.

1
i
2

t
Sol. curve 1

9.2 Decay of Current


Let the circuit be disconnected from battery and switch S is thrown to point b in the figure.
The current now begins to fall. In the absence of inductance, the current would have fallen
from maximum i0 to zero almost instantaneously. But due to the presence of inductance,
which opposes the decay of current, the rate of decay of current is reduced.
suppose during the decay of current, i be the value of current at any instant t. Using
Kirchhoff’s voltage law in the circuit, we get
di di R
–L  Ri or – i
dt dt L i
Integrating this expression, we get
R R
loge i = – t + B L
L
where B is constant of integration. The value of B can be obtained by applying the condition
that when t = 0, i = i0
 loge i0 = B
Substituting the value of B, we get
R
logei = – t + logei0
L
i R
or loge i =– t
0 L

 R 
or (i / i0) = exp  – L t
 ...(6)

 R   t
or i  i0 exp   t  i0 exp   
 L   
where  = L/R = inductive time constant of the circuit.
It is obvious from equation that the current in the circuit decays exponentially as shown in
figure.

394,50 - Rajeev Gandhi Nagar Kota, Ph. No. : 93141-87482, 0744-2209671

Downloaded from www.iitjeephysics4u.com


IVRS No : 0744-2439051, 52, 53, www. motioniitjee.com, info@motioniitjee.com
ELECTROMAGNETIC INDUCTION Page # 31

• We observe the following points


(i) After t = L/R, the current in the circuit is given by i0

i Decay of current
 R L
i = i0 exp  – L  R  = i0 exp(–1)

= (i0 / e) = i0/2.718 = 0.37 i0 O t


So after a time (L/R) second, the current reduces to 37% of the maximum current i0. (L/R) is
known as time constant . This is defined as the time during which the current decays to 37%
of the maximum current during decay.
(ii) The rate of decay of current in given by

di d   R 
= dt i0 exp  – L t 

dt 

di R  R 
t = – R i
 = i exp  – L  ...(7)
dt L 0 L

di R
or – = i
dt L
This equation shows that when L is small, the rate of decay of current will be large i.e., the
current will decay out more rapidly.

Ex.46 In the following circuit the switch is closed at t = 0. Initially there is no current in
inductor. Find out current the inductor coil as a function of time.
R R

t=0
R L

Sol. At any time t


–  + i1 R – (i – i1) R = 0
–  + 2i1 R – i R = 0 R R
iR   – +
i1R +

i1 = t=0 – iR –
2R di
(i–i1)R R L
di  + dt
Now, – + i1 R + iR +L. =0 i–i1 +
dt
i1 i
 iR    di  3 IR di
–  +   + iR + i. 0 –   –L
2  dt 2 2 dt
 –  3 iR  –di di
  dt = – L. di  – 
2 dt –  3iR
t i
dt di t 1  –  3iR 
–    –  3iR  –  ln
0
2L 0
2L 3R  – 

 –  3iR  3Rt
– ln  – 
=
2L
3Rt
  –
2L

i = + 3R  1 – e 

394,50 - Rajeev Gandhi Nagar Kota, Ph. No. : 93141-87482, 0744-2209671

Downloaded from www.iitjeephysics4u.com


IVRS No : 0744-2439051, 52, 53, www. motioniitjee.com, info@motioniitjee.com
Page # 32 ELECTROMAGNETIC INDUCTION

Ex.47 Figure shows a circuit consisting of a ideal cell, an inductor L and a resistor R, connected
in series. Let the switch S be closed at t = 0. Suppose at t = 0 current in the inductor is
i0 then find out equation of current as a function of time
R i0 L
C

B S  A
Sol. Let an instant t current in the circuit is i which is increasing at the rate di/dt.
Writing KVL along the circuit, we have
di di R L
– L – iR = 0  L =  – iR
dt dt
C
i
di
t
dt   – iR  i
Rt
   – iR  
i0 0
L  ln   – i R  = –
  L
0
B S  A
 – ( – i0R)e–Rt / L
  – iR = ( – i0R)e–Rt/L i=
R

10. MUTUAL INDUCTANCE

Consider two coils P and S placed close to each other as shown in the figure. When the
current passing through a coil increases or decreases, the magnetic flux linked with the other
coil also changes and an induced e.m.f. is developed in it. This phenomenon is known as
mutual induction. This coil in which current is passed is known as primary and the other in
which e.m.f. is developed is called as secondary.
Let the current through the primary coil at any instant be i1. Then the magnetic flux 2 in the
secondary at any time will be proportional to i1 i.e., 2  i1
Therefore the induced e.m.f. in secondary
i1
when i1 changes is given by P

d2 di1
  i.e.,   
dt dt
S
di dMi1
   M 1 = –  2 = M i1
dt dt
where M is the constant of proportionality and is known as mutual inductance of two coils. It
is defined as the e.m.f. induced in the secondary coil by unit rate of change of current in the
primary coil. The unit of mutual inductance is henry (H).

10.1 Mutual Inductance of a Pair of Solenoids one Suurounding the other coil
Figure shows a coil of N2 turns and radius R2 surrounding a long solenoid of length l1, radius R1
and number of turns N1.
S2
S1
To calculate mutual inductance M between them, let us assume a current i1 through the inner
solenoid S1
There is no magnetic field outside the solenoid and the field inside has magnitude,

394,50 - Rajeev Gandhi Nagar Kota, Ph. No. : 93141-87482, 0744-2209671

Downloaded from www.iitjeephysics4u.com


IVRS No : 0744-2439051, 52, 53, www. motioniitjee.com, info@motioniitjee.com
ELECTROMAGNETIC INDUCTION Page # 33

N 
B  0  1  i1
 l1 
and is directed parallel to the solenoid’s axis. The magnetic flux B2 through the surrounding
coil is, therefore,
0N1 i1
B2  B(R12 )  R12
l1

N2 B2 N    N i   N N R 2
Now, M   2   0 1 1  R12  0 1 2 1
i1  i1   l1  l1
Notice that M is independent of the radius R2 of the surrounding coil. This is because solenoid’s
magnetic field is confined to its interior.
Brain Teaser
What is the meaning of the statement “The coefficient of mutual inductance for a pair of
coils is large” ?

Note : M  L1L 2
For two coils in series if mutual inductance is considered then
Leq = L1 + L2 ± 2M

Ex.48 Find the mutual inductance of two concentric coils of radii a1


and a2
(a1 << a2) if the planes of coils are same. a2
Sol. Let a current i flow in coil of radius a2. a1
 0i 2
Magnetic field at the centre of coil = 2a a1
2
2
 0i 2  0 a1
or M i = 2a a1 or M 
2 2a 2

Ex.49 Solve the above question, if the planes of coil are perpendicular.
Sol. Let a current i flow in the coil of radius a1. The magnetic field at the centre of this coil will
now be parallel to the plane of smaller coil and hence no flux will pass through it, hence M =
0

Ex.50 Solve the above problem if the planes of coils make  angle with each other.
Sol. If i current flows in the larger coil, magnetic field produced at the centre will be perpendicular
to the plane of larger coil.
Now the area vector of smaller coil which is perpendicular to the plane of smaller coil will make
an angle  with the magnetic field.
   0i
Thus flux = B.A  2a . a12 cos 
2

2
0 a cos 1
1
or M=
2a2
b
Ex.51 Find the mutual inductance between two rectangular b
loops, shown in figure.
Sol. Let current i flow in the loop having -by long a c
sides. Consider a segment of width dx at a distance
x as shown flux through the regent a
b
i
  0i 0i  i x b
d =  –  b dx
 2x 2(x  a)  c dx

394,50 - Rajeev Gandhi Nagar Kota, Ph. No. : 93141-87482, 0744-2209671

Downloaded from www.iitjeephysics4u.com


IVRS No : 0744-2439051, 52, 53, www. motioniitjee.com, info@motioniitjee.com
Page # 34 ELECTROMAGNETIC INDUCTION

c b
  0i 0i 
 =   2x – 2(x  a)  b dx
c  
 0ib  c  b a  b  c
= 2 ln c – ln a  c 
 

Ex.52 Figure shows two concentric coplanar coils


with radii a and b (a << b). A current i = 2t b
flows in the smaller loop. Neglecting self
a
inductance of larger loop
(a) Find the mutual inductance of the two coils
(b) Find the emf induced in the larger coil
(c) If the resistance of the larger loop is R find the current in it as a function of time
Sol. (a) To find mutual inductance, it does not matter in which coil we consider current and in
which flux is calculated (Reciprocity theorem) Let current i be flowing in the larger coil.
0i
Magnetic field at the centre = .
2b

0i 2
flux through the smaller coil = a
2b
0
 M= a2
2b
 di  
(ii) | emf induced in larger coil | = M  dt  in smaller coil
 

0  a2
= a2 (2) = 0
2b b

 0 a2
(iii) current in the larger coil =
bR
Ex.53 If the current in the inner loop changes according to i = 2t2
C
then, find
the current in the capacitor as a function of time.

0
Sol. M= a 2
2b
 di  
q–q
| emf induced in larger coil | = M  dt  in smaller coil
 
i
0 20 a t 2
R
e= a2 (4t) =
2b b
Applying KVL : - e

q 20 a2 t q
+e – – iR = 0  – –iR = 0
c b c

2 0 a2 i di 2 0 a2C
differentiate wrt time : – – R  0 on solving it i = 1 – e–t / RC 
 
b c dt b

394,50 - Rajeev Gandhi Nagar Kota, Ph. No. : 93141-87482, 0744-2209671

Downloaded from www.iitjeephysics4u.com


IVRS No : 0744-2439051, 52, 53, www. motioniitjee.com, info@motioniitjee.com
ELECTROMAGNETIC INDUCTION Page # 35

11. SERIES COMBINATION OF INDUCTORS

i L1 L2 V
O V = O V
V1 V2 Leq

 V = V1 + V2

di di di
L eq  L1  L2
dt dt dt
Leq = L1 + L2 +.................
Parallel Combination of inductor

di di di i1
i = i1 + i2   1  2
dt dt dt
O i V
v v v
 
L eq L1 L 2
i2
1 1 1
   ...........
L eq L1 L 2

394,50 - Rajeev Gandhi Nagar Kota, Ph. No. : 93141-87482, 0744-2209671

Downloaded from www.iitjeephysics4u.com


IVRS No : 0744-2439051, 52, 53, www. motioniitjee.com, info@motioniitjee.com
Page # 36 ELECTROMAGNETIC INDUCTION

Exercise - I (OBJECTIVE PROBLEMS )

SECTION (A) : FLUX AND FARADAY’S LAWS OF 4. A square wire loop of 10.0 cm side lies at right
ELECTROMAGNETIC INDUCTION & LENZ’S LAW angles to a uniform magnetic field of 20T. A 10 V
1. A conducting loop of radius R is present in a light bulb is in a series with the loop as shown in
uniform magnetic field B perpendicular the plane the fig. The magnetic field is decreasing steadily
of the ring. If radius R varies as a function of time to zero over a time interval t. The bulb will shine
‘t’, as R = R0 + t. The e.m.f induced in the loop is with full brightness if t is equal to
× × × × × × × × ×
× × × × × × × × × × × × × × × ×
× × × × × × × × × × × × × × × ×
× × × × × × × × × × × × × × × ×
× × B× × × R × × × × × × × × ×
× × × × × × × × × × × × × × × ×
× × × × × × × × × × × × × × × ×
× × × × × × × × × × × × × × × ×
(A) 20 ms (B) 0.02 ms
(A) 2 (R0 + t) B clockwise (C) 2 ms (D) 0.2 ms
(B) (R0 + t)B clockwise
5. A long straight wire is parallel to one edge as
(C) 2(R0 + t)B anticlockwise
in fig. If the current in the long wire is varies in
(D) zero
time as I = I0e–t /, what will be the induced emf in
the loop ?
2. An electron is moving in a circular orbit of i
radius R with an angular acceleration . At the d
centre of the orbit is kept a conducting loop of
a
radius r, (r < < R). The e.m.f induced in the smaller
loop due to the motion of the electron is b
(A) zero, since charge on electron in constant
 0 er 2  0 bI  d  a   0 bI  d  a 
(B)  (A) ln  (B) ln 
4R   d  2  d 
 er 2
(C) 0  2 0 bI  d  a   0 bI  d 
4 R l n  ln 
(C)  d  (D)
(D) none of these    d  a 
3. A wire loop is placed in a region of time varying
magnetic field which is oriented orthogonally to 6. The magnetic flux through a stationary loop
the plane of the loop as shown in the figure. The with resistance R varies during interval of time T
graph shows the magnetic field variation as the as  = at (T – t). The heat generated during this
function of time. Assume the positive emf is the time neglecting the inductance of loop will be
one which drives a current in the clockwise
direction and seen by the observer in the direction a2 T 3 a2 T 2
(A) (B)
of B. Which of the following graphs best represents 3R 3R
the induced emf as a function of time.
a2 T a2 T 3
B (C) (D)
3R R
t1 t2 t1 t
7. The dimensions of permeability of free space
  can be given by
(A) [MLT–2A–2] (B) [MLA–2]
t1 t2 t1
t1 t2 t t –3 2 2
(C) [ML T A ] (D) [MLA–1]
(A) t1 (B)

8. A closed planar wire loop of area A and arbitrary


shape is placed in a uniform magnetic field of
magnitude B, with its plane perpendicular to
 
magnetic field. The resistance of the wire loop is
t1 t2 t1 R. The loop is now turned upside down by 180°
t t
(C) t2 t1 (D) t1 so that its plane again becomes perpendicular to
the magnetic field. The total charge that must

394,50 - Rajeev Gandhi Nagar Kota, Ph. No. : 93141-87482, 0744-2209671

Downloaded from www.iitjeephysics4u.com IVRS No : 0744-2439051, 52, 53, www. motioniitjee.com, info@motioniitjee.com
ELECTROMAGNETIC INDUCTION Page # 37

have flowen through the wire ring in the process from ‘c’ to ‘d’ in loop (2)
is
(A) < AB/R (B) = AB/R
(C) = 2AB/R (D) None loop (1)
a b
9. A square coil ABCD is placed in x-y plane with
its centre at origin. A long straight wire, passing loop (2)
through origin, carries a current in negative z- c d
direction. Current in this wire increases with
time.The induced current in the coil is In an experiment, the graph of i2 against time ‘t’
y
B C
is as shown below
 i2
x
0
A D t
Which one(s) of the following graphs could have
(A) clockwise (B) anticlockwise
caused i2 to behave as give above.
(C) zero (D) alternating
i1 i1
10. A vertical bar magnet is dropped from position
(A) (B)
on the axis of a fixed metallic coil as shown in 0 0
fig-I. In fig-II the magnet is fixed and horizontal t t
coil is dropped. The acceleration of the magnet
i1 i1
and coil are a1 and a2 respectively then t
0
S (C) (D)
0
t
S
N i1
fixed fixed N (E) t
fig-1 fig-II 0
(A) a1 > g, a2 > g (B) a1 > g, a2 < g
(C) a1 < g, a2 < g (D) a1 < g, a2 > g 14. Figure shows a bar magnet and a long straight
wire W, carrying current into the plane of paper.
11. Two identical coaxial circular loops carry a Point P is the point of intersection of axis of
current i each circulating in the same direction. magnet and the line of shortest distance between
If the loops approach each other magnet and the wire. If p is the midpoint of the
(A) the current in each will decrease magnet, then which of the following statements
(B) the current in each will increase is correct ? P
(C) the current in each will remain the same N S
(D) the current in one will increase and in other
will decrease

12. In the arrangement shown in given figure
W
current from A to B is increasing in magnitude.
Induced current in the loop will (A) magnet experiences a torque in clockwise
direction
(B) magnet experiences a torque in anticlockwise
direction
(C) magnet experiences a force, normal to the
A B
(A) have clockwise direction line of shortest distance
(B) have anticlockwise direction (D) magnet experiences a force along the line of
(C) be zero shortest distance
(D) oscillate between clockwise and anticlockwise 15. A negative charge is given to a nonconducting
13. An electric current i1 can flow either direction loop and the loop is rotated in the plane of paper
through loop (1) and induced current i2 in loop about its centre as shown in figure. The magnetic
(2). Positive i1 is when current is from ‘a’ to ‘b’ in field produced by the ring affects a small magnet
loop (1) and positive i2 is when the current is placed above the ring in the same plane :

394,50 - Rajeev Gandhi Nagar Kota, Ph. No. : 93141-87482, 0744-2209671

Downloaded from www.iitjeephysics4u.com


IVRS No : 0744-2439051, 52, 53, www. motioniitjee.com, info@motioniitjee.com
Page # 38 ELECTROMAGNETIC INDUCTION

velocity v perpendicular to one of its sides. A


uniform and constant magnetic field B exists along
S N
the perpendicular to the plane of the loop in fig.
The current induced in the loop is
× × × × × ×

(A) the magnet does not rotate × × × × × ×


(B) the magnet rotates clockwise as seen from v
× × × × × ×
below.
× × × × × ×
(C) the magnet rotates anticlockwise as seen (A) BI/R clockwise (B) BI/R anticlockwise
from below (C) 2BI/R anticlockwise (D) zero
(D) no effect on magnet is there.

16. Two conducting rings P and Q of radii r and 2r 20. An equilateral triangular loop ADC having some
rotate uniformly in opposite directions with centre resistance is pulled with a constant velocity v
of mass velocities 2v and v respectively on a out of a uniform magnetic field directed into the
conducting surface S. There is a uniform magnetic paper. At time t = 0, side DC of the loop is at
field of magnitude B perpendicular to the plane of edge of the magnetic field. The induced current
the rings. The potential difference between the (i) versus time (t) graph will be as
× × × × ×
highest points of the two rings is A
2v B v × × × × ×
r 2r
× × × × ×
P S Q
(A) zero (B) 4 Bvr D C
(C) 8 Bvr (D) 16 Bvr × × v× × ×
i i
17. The magnetic field in a region is given by
  x (A) (B)
B  B 0  1   k . A square loop of edge-length d is
 a
placed with its edge along x & y axis. The loop is t t

moved with constant velocity V  V0 i . The emf i i
induced in the loop is
V0B 0 d2 V0B 0 d2 (C) (D)
(A) (B)
a 2a
t t
V0B 0 a 2
(C) (D) None
d
21. A semicircular conducting wire is placed in yz
18. Consider the situation shown in fig. The plane in a uniform magnetic field directed along
resistanceless wire AB is slid on the fixed rails positive z-direction. An induced emf will be
with a constant velocity. If the wire AB is replaced developed between the ends of the wire if it is
by a resistanceless semicircular wire, the moved along. z
magnitude of the induced current will
× × ×A × B

× ×v y
× × × ×
× × × B× x
(A) increase (B) remain the same (A) positive x direction
(C) decrease (B) positive y direction
(D) increase or decrease depending on whether (C) positive z direction
the semicircle bulges towards the resistance or (D) none of these
away from it.
22. Two circular coils A and B are facing each
19. A conducting square loop of side I and other as shown in figure.
resistance R moves in its plane with a uniform

394,50 - Rajeev Gandhi Nagar Kota, Ph. No. : 93141-87482, 0744-2209671

Downloaded from www.iitjeephysics4u.com IVRS No : 0744-2439051, 52, 53, www. motioniitjee.com, info@motioniitjee.com
ELECTROMAGNETIC INDUCTION Page # 39

A B
SECTION (B) : E.M.F. INDUCED IN A MOVING
ROD
25. A rectangular loop with a sliding connector of
length 10 cm is situated in uniform magnetic field
perpendicular to plane of loop. The magnetic
induction is 0.1 tesla and resistance of connector
i
(R) is 1 ohm. The sides AB and CD have resistances
~
2 ohm and 3 ohm respectively. Find the current
The current i through A can be altered in the connector during its motion with constant
(A) there will be repulsion between A and B if i is velocity one meter/sec.
increased D
A
(B) there will be attraction between A and B if i is
increased 2 3
(C) there will be neither attraction nor repulsion R
when i is changed B C
(D) attraction or repulsion between A and B 1 1 1 1
depends on the direction of current. (A) A (B) A (C) A (D) A
110 220 55 440
It does not depend whether the current increased
or decreased 26. Two infinitely long conducting parallel rails
are connected through a capacitor C as shown in
23. Two identical conductors P and Q are placed the figure. A conductor of length l is moved with
on two frictionless fixed conducting rails R and S constant speed v0. Which of the following graph
truly depicts the variation of current through the
in a uniform magnetic field directed into the plane.
conductor with time ?
If P is moved in the direction shown in figure with
 B 
a constant speed, then rod Q
P Q  
B v0
l
 
× × ×
R  
V
× × ×
S Current Current
× × × I (t) I (t)
(A) (B)
t (time) t (time)
(A) will be attracted towards P
(B) will be repelled away from P
(C) will remain stationary
Current Current
(D) may be repelled or attracted towards P I (t) I (t)
(C) (D)
24. A square coil ACDE with its plane vertical is
I = 0 t (time) t (time)
released from rest in a horizontal uniform magnetic

field B of length 2L. The acceleration of the coil is
C D 27. The figure shows an isosceles triangle wire
L frame with apex angle equal to /2. The frame
A E starts entering into the region of uniform magnetic
B
× × × × field B with constant velocity v at t = 0. The
× × × × longest side of the frame is perpendicular to the
2L
× × × × direction of velocity. If i is the instantaneous
× × × × current through the frame then choose the
(A) less than g for all the time till loop crosses alternative showing the correct variation of i with
the magnetic field completely time.
(B) less than g when it enters the field and greater × B× ×
× × ×
than g when it comes out of the field
(C) g all the time 90° × ×v ×
× × ×
(D) less than g when it enters and comes out of × × ×
the field but equal to g when it is within the field × × ×
t=0

394,50 - Rajeev Gandhi Nagar Kota, Ph. No. : 93141-87482, 0744-2209671

Downloaded from www.iitjeephysics4u.com


IVRS No : 0744-2439051, 52, 53, www. motioniitjee.com, info@motioniitjee.com
Page # 40 ELECTROMAGNETIC INDUCTION

i i  0 vI l  0 vI l
(A) (B)
r 2 r
(A) (B) 2 0 v Il  0 v Il
(C) (D)
t t r 4 r

i i 32. A conducting rod of length l moves with


velocity  a direction parallel to a long wire carrying
(C) (D) a steady current I. The axis of the rod is
t t maintained perpendicular to the wire with near
end a distance raway as shown in the fig. Find
28. A thin wire of length 2m is perpendicular to
the emf induced in the rod.
the xy plane. It is m oved with ve locity

v  (2 i  3 j  k ) m / s through a region of magnetic I

induction B  ( i  2j ) Wb/m 2 . Then potential
r
difference induced between the ends of the wire
(A) 2 volts (B) 4 volts
(C) 0 volts (D) none of these l 
29. A long metal bar of 30 cm length is aligned
along a north south line and moves eastward at
a speed of 10 ms–1. A uniform magnetic field of  0I  r  l  2 0I  r  l 
(A) ln  (B) ln 
4.0 T points vertically downwards. If the south   r    r 
end of the bar has a potential of 0V, the induced
potential at the north end of the bar is  0I  l   0I  r + l 
(A) + 12 V (B) – 12 V (C) ln  (D) ln 
 r  l 2  r 
(C) 0 V
(D) cannot be determined since there is not closed 33. A square loop of side a and resistance R is
circuit moved in the region of uniform magnetic field B
30. A square metal loop of side 10 cm and (loop remaining completely insidefield), with a
resistance 1  is moved with a constant velocity velocity v through a distance x. The work done is
partly inside a magnetic field of 2 Wbm–2, directed Bl 2 vx 2B 2 l 2 vx
(A) (B)
into the paper, as shown in the figure. This loop R R
is connected to a network of five resistors each 2 2
4B l vx
of value 3 . If a steady current of 1 mA flows in (C) (D) none
R
the loop, then the speed of the loop is 34. A metallic rod of length L and mass M is moving
× × × × v under the action of two unequal forces F1 and F2
× × × × (directed opposite to each other) acting at its
× × × ×
× × × ×
ends along its length. Ignore gravity and any
× × × × external magnetic field. If specific charge of
× × × × electrons is (e/m), then the potential difference
(A) 0.5 cms–1 (B) 1 cms–1 between the ends of the rod is steady state must
(C) 2 cms–1 (D) 4 cms–1 be
(A) |F1 – F2| mL/eM (B) (F1 – F2) mL/eM
31. A conducting rod moves with constant velocity (C) [mL/eM] ln [F1/F2] (D) None
v perpendicular to the long, straight wire carrying
a current I as shown compute that the emf 35. A rod closing the circuit shown in figure moves
generated between the ends of the rod. along a U shaped wire at a constant speed v
under the action of the force F. The circuit is in a
uniform magnetic field perpendicular to the plane.
r Calculate F if the rate of heat generation in the
I circuit is Q.

l v
× × × × ×
× × × × ×
× × × × ×
× × × ×F'×
× × × × ×
× × × × ×

394,50 - Rajeev Gandhi Nagar Kota, Ph. No. : 93141-87482, 0744-2209671

Downloaded from www.iitjeephysics4u.com IVRS No : 0744-2439051, 52, 53, www. motioniitjee.com, info@motioniitjee.com
ELECTROMAGNETIC INDUCTION Page # 41

Q
A H K
(A) F = Qv (B) F = × 
× × × ×
v B
v
R × × × × ×
(C) F  (D) F  Qv × ×× × ×C
× ×V ×
Q
× ×D
36. Two parallel long straight conductors lie on a
G E
(A) clockwise (B) anticlockwise
smooth surface. Two other parallel conductors (C) alternating (D) Zero
rest on them at right angles so as to form a
square of side a initially. A uniform magnetic field 40. A conducting rod is moved with a constant
B exists at right angles to the plane containing 
velocity  in a magnetic field. A potential
the conductors. They all start moving out with a
difference appears across the two ends
constant velocity v. If r is the resistance per unit    
length of the wire the current in the circuit will (A) if || l (B) if ||B
 
be (C) if l ||B (D) none of these
Bv Br
(A) (B)
r v 41. A uniform magnetic field exists in region given
(C) B v r (D) Bv 
by B  3 i  4 j  5k . A rod of length 5 m is placed
along y-axis is moved along x-axis with constant
37. There is a uniform magnetic field B normal to
speed 1 m/sec. Then induced e.m.f. in the rod
the xy plane. A conductor ABC has length AB =
will be
l1, parallel to the x-axis, and length BC = l2,
(A) zero (B) 25 v
parallel to the y-axis. ABC moves in the xy plane
(C) 20 v (D) 15v
with velocity v x i  v y j . The potential difference
42. BACD is a fixed conducting smooth rail placed
between A and C is proportional to
in a vertical plane. PQ is a conducting rod which
C is free to slide on the rails. A horizontal uniform
magnetic field exists in space as shown. If the
j
A rod PQ is released from rest then,
B A C
i

(A) vxl1 + vy l2 (B) vxl2 + vy l1


(C) vxl2 – vy l1 (D) Vxl1 – vy l2 P Q

38. A conducting rod PQ of length 5 m oriented


as shown in figure is moving w it h v e l o c i t y
B D
(2m / s)i without rotation in a uniform magnetic (A) The rod PQ will move downward with constant
field ( 3 j  4k ) Tesla. Emf induced in the rod is acceleration
(B) The rod PQ will move upward with constant
y
Q acceleration
2m/s (C) The rod will move downward with decreasing
acceleration and finally acqcure a constant
5m
velocity
53°
(D) either A or B
P x
(A) 32 Volts (B) 40 Volt 43. A cylindrical space of radius R is filled with a
(C) 50 Volt (D) none uniform magnetic induction B parallel to the axis
of the cylinder. If B changes at a constant rate,
39. In the circuit shown in figure, a conducting the graph showing the variation of induced electric
wire HE is moved with a constant speed V towards field with distance r from the axis of cylinder is
left. The complete circuit is placed in a uniform × × ×
 × × × × ×
magnetic field B perpendicular to the plane of
the circuit directed in inward direction. The current × × × × ×
R
in HKDE is × × ×

394,50 - Rajeev Gandhi Nagar Kota, Ph. No. : 93141-87482, 0744-2209671

Downloaded from www.iitjeephysics4u.com


IVRS No : 0744-2439051, 52, 53, www. motioniitjee.com, info@motioniitjee.com
Page # 42 ELECTROMAGNETIC INDUCTION

field, then the average induced e.m.f. when it


E E
has rotated through 90° is
BA BA
(A) (B)
(A) (B)  2
r r BA 2BA
(C) (D)
4 
E E

47. A ring of resistance 10, radius 10cm and


100 turns is rotated at a rate 100 revolutions per
(C) (D)
second about its diameter is perpendicular to a
r r uniform magnetic field of induction 10 mT. The
amplitude of the current in the loop will be nearly
SECTION (C) : INDUCED EMF IN ROD, RING DISC (Take : 2 = 10)
ROTATING IN A UNIFORM MAGNETIC FIELD (A) 200 A (B) 2A
44. When a ‘J’ shaped conducting rod is rotating (C) 0.002 A (D) none of these
in its own plane with constant angular velocity
, about one of its end P, in a uniform magnetic 48. A copper rod AB of length L, pivoted at one
 end A, rotates at constant angular velocity , at
field B directed normally into the plane of paper)
then magnitude of emf induced across it will be right angles to a uniform magnetic field of
× × × × × × × × induction B. The e.m.f developed between the
× × × × × × × × mid point C of the rod and end B is
× × × × × × × ×
× × ×l × × ×L × × × ×
× × × × × × ×Q × × × ×
× × × × × × × × × × ×× L ×
× P× × × × × × × × A B
× × × × ×
× × × × × × × × × ×
× × × × ×
1
(A) B L2  l 2 (B) BL2 × ×
2
Bl 2 Bl 2
1 2 2 1 2 (A) (B)
(C) B(L  l ) (D) Bl 4 2
2 2
3Bl 2 3Bl 2
45. A metal disc rotates freely, between the poles (C) (D)
4 8
of a magnet in the direction indicated. Brushes P
and Q make contact with the edge of the disc 49. A rod of length l rotates with a uniform angular
and the metal axle. What current, if any, flows velocity  about its perpendicular bisector. A
through R? uniform magnetic field B exists parallel to the axis
N of rotation. The potential difference between the
two ends of the rod is
R 1 2
(A) zero (B) B (C) B  2 (D) 2B  2
P  2
Q

S 50. A rod of length 10 cm made up of conducting


and non-conducting material (shaded part is non-
(A) a current from P to Q
conducting). The rod is rotated with constant
(B) a current from Q to P
angular velocity 10 rad/sec about point O, in
(C) no current, because the emf in the disc is
constant magnetic field of 2 tesla as shown in
opposed by the back emf
the figure. The induced emf between the point A
(D) no current, because the emf induced in one
and B of rod will be
side of the disc is opposed by the emf induced in × × × × ×
the other side. 3cm A
(E) no current, because no radial emf is induced × × × × ×
in the disc. × × × × ×
B
× × × × ×
46. A rectangular coil of single turn, having area 2cm
A, rotates in a uniform magnetic field B an angular × × × × ×
velocity  about an axis perpendicular to the field.
If initially the plane of coil is perpendicular to the × O × × × ×

394,50 - Rajeev Gandhi Nagar Kota, Ph. No. : 93141-87482, 0744-2209671

Downloaded from www.iitjeephysics4u.com


IVRS No : 0744-2439051, 52, 53, www. motioniitjee.com, info@motioniitjee.com
ELECTROMAGNETIC INDUCTION Page # 43

(A) 0.029 V (B) 0.1 V 54. For L - R circuit, the time constant is equal
(C) 0.051 V (D) 0.064 V to
(A) twice the ratio of the energy stored in the
51. A uniform magnetic field of induction B is magnetic field to the rate of dissipation of energy
confined to a cylindrical region of radius R. The in the resistance.
magnetic field is increasing at a constant rate of (B) ratio of the energy stored in the magnetic
dB field to the rate of dissipation of energy in the
(tesla/second). An electron of charge q, resistance.
dt
placed at the point P on the periphery of the field (C) half the ratio of the energy stored in the
experiences an acceleration magnetic field to the rate of dissipation of energy
in the resistance
× × × (D) square of the ratio of the energy stored in
× × R ×
× × × × × the magnetic field to the rate of dissipation of
× × ×× × energy in the resistance.
× × × × ×
× × × ×
× × × 55. In the adjoining circuit, initially the switch S is
P open. The switch ‘S’ is closed at t = 0. The
difference between the maximum and minimum
1 eR dB 1 eR dB current that can flow in the circuit is
(A) toward left (B) toward right
2 m dt 2 m dt
10V s
eR dB ()
(C) toward left (D) zero 0.1 H 10
m dt

52. Figure shows a uniform magnetic field B 10


confined to a cylindrical volume and is increasing
at a co nstant rate. The ins tantaneo us (A) 2 Amp (B) 3 Amp
acceleration experienced by an electron placed at (C) 1 Amp
P is (D) nothing can be concluded
× × P ×
× ×
× × × 56. The ratio of time constant in charging and
× × × × discharging in the circuit shown in figure is
× ×× ×
× ×
R
(A) zero (B) towards right L
(C) towards left (D) upwards
2R
53. A triangular wire frame (each side = 2m) is
placed in a region of time variant magnetic field V
having dB/dt = 3 T / s . The magnetic field is (A) 1 : 1 (B) 3 : 2
perpendicular to the plane of the triangle. The (C) 2 : 3 (D) 1 : 3
base of the triangle AB has a resistance 1  while
the other two sides have resistance 2 each. 57. In an L-R circuit connected to a battery of
The magnitude of potential difference between constant e.m.f E switch S is closed at time t = 0.
the points A and B will be If e denotes the magnitude of induced e.m.f across
inductor and i the current in the circuit at any
×C time t. Then which of the following graphs shows
×× × ××
×× ×× ××× the variation of e with i ?
××× × ××
× ×× ×× e e
×
× ×× ××
× ××
×
A× × × × × B
×××× (A) (B)
i i
(A) 0.4 V (B) 0.6 V
(C) 1.2 V (D) None e e

SECTION (D) :
SELF INDUCTANCE, MUTUAL INDUCTANCE, (C) (D)
CIRCUIT CONTAINING RESISTANCE i i
INDUCTANCE, AND L.C. OSCILLATIONS

394,50 - Rajeev Gandhi Nagar Kota, Ph. No. : 93141-87482, 0744-2209671

Downloaded from www.iitjeephysics4u.com


IVRS No : 0744-2439051, 52, 53, www. motioniitjee.com, info@motioniitjee.com
Page # 44 ELECTROMAGNETIC INDUCTION

58. A current of 2A is increasing at a rate of 4A/ 64. A long straight wire of circular cross-section
s through a coil of inductance 2H. The energy is made of a non-magnetic material. The wire is
stored in the inductor per unit time is of radius a. The wire carries a current I which is
(A) 2J/s (B) 1 J/s uniformly distributed over its cross-section. The
(C) 16 J/s (D) 4 J/s energy stored per unit length in the magnetic
field contained within the wire is :
59. Two identical inductance carry currents that
vary with time according to linear laws (as shown  0I2  0I2
(A) U = (B) U 
in figure). In which of two inductance is the self 8 16 
induction emf greater ?
 0I2  0I2
(C) U  (D) U 
4 2
I
65. The network shown in the figure is part of a
2
1 complete circuit. If at a certain instant, the
current I is 5A and it is decreasing at a rate of
t
103 As–1 then VB – VA equals
(A) 1 (B) 2 1 + 5 mH
(C) same A B
(D) data are insufficient to decide I
15 V
(A) 20 V (B) 15 V
60. The current in the given circuit is increasing
(C) 10 V (D) 5 V
with a rate a = 4 amp/s. The charge on the
capacitor at an instant when the current in the
66. In Previous Problem if I is reversed in
circuit is 2 amp will be :
direction, then VB – VA equals
E=4V (A) 5V (B) 10 V
(C) 15 V (D) 20 V

R  1 67. Two resistors of 10 , and 20  and an ideal


inductor of 10 H are connected to a 2 V battery
as shown. The key K is inserted at time t = 0.
L=1H C  3 F The initial (t = 0) and final (t ) currents through
battery are
(A) 4 C (B) 5 C
10 H
(C) 6C (D) none of these

61. L, C and R represent physical quantities


inductance, capacitance and resistance. The 10 20
combination which has the dimensions of
frequency is K

1 R 1 R
(A) and (B) and 2V
RC L RC L 1 1 1 1
(A) A, A (B) A, A
C 15 10 10 15
(C) LC (D)
L
2 1 1 2
(C) A, A (D) A, A
62. A coil of inductance 5H is joined to a cell of 15 10 15 25
emf 6V through a resistance 10 at time t = 0.
The emf across the coil at time t = ln 2 s is : 68. A small coil of radius r is placed at the centre
of a large coil of radius R, where R >> r. The coils
(A) 3 V (B) 1.5 V
are coplanar. The coefficient of mutual inductance
(C) 0.75 V (D) 4.5 V
between the coils is
 r  r 2
63. A long solenoid of N turns has a self inductance (A) 0 (B) 0
L and area of cross section A. When a current i 2R 2R
2
flows through the solenoid, the magnetic field  0 r  0 r
(C) (D)
inside it has magnitude B. The current i is equal 2R 2
2R 2
to :
(A) BAN/L (B) BANL (C) BN/AL (D) B/ANL

394,50 - Rajeev Gandhi Nagar Kota, Ph. No. : 93141-87482, 0744-2209671

Downloaded from www.iitjeephysics4u.com


IVRS No : 0744-2439051, 52, 53, www. motioniitjee.com, info@motioniitjee.com
ELECTROMAGNETIC INDUCTION Page # 45

69. Two long parallel wires whose centres are a 73. In a L - R decay circuit, the initial current at
distance d apart carry equal currents in opposite t = 0 is I. The total charge that has flown through
directions. If the flux within wires is neglected, the resistor till the energy in the inductor has
the inductance of such arrangement of wire of reduced to one-fourth its initial value, is
length l and radius a will be (A) L I / R (B) L I / 2R
 0l d– a  l d (C) L I 2 / R (D) None
(A) L = loge (B) L = 0 loge
 a  a
74. An inductor coil stores U energy when i current
 l a
(C) L = 0 loge (D) none is passed through it and dissipates energy at the
 d rate of P. The time constant of the circuit, when
70. In the circuit shown, the cell is ideal. The coil this coil is connected across a battery of zero
has an inductance of 4H and zero resistance. F is internal resistance is
a fuse of zero resistance and will blow when the 4U U 2U 2P
current through it reaches 5A. The switch is closed (A) (B) (C) (D)
P P P U
at t = 0. The fuse will blow :
L
75. The mutual inductance between the
fuse rectangular loop and the long straight wire as
SW shown in figure is M.
b
2V
(A) just after t = 0 (B) after 2s 
B
(C) after 5 s (D) after 10s i1 c
a
71. In the LR circuit shown, what is the variation
of the current I as a function of time ? The switch
is closed at time t = 0 sec.
I 2V 0a  c
(A) M = Zero (B) M = ln 1  
2  b
L R V
 0b  a  c   0a  b
(C) M  ln  (D) M  ln 1  
2  b  2  c
R
3V 76. A long straight wire is placed along the axis
Rt 
V  L V
Rt of a circular ring of radius R. The mutual
(A) R  1– e
 –
 (B) e L inductance of this system is
  R
 0R  0 R
(A) (B)
Rt 2 2
V –L
(C) – e (D) None 0
R (C) (D) 0
2
72. In the circuit shown, X is joined to Y for a L
long time, and then x is joined to Z. The total 77. In the circuit shown in the figure, R = .
C
heat produced in R2 is : Switch S is closed at time t = 0. The current
through C and L would be equal after a time t
R2
equal to :
X Z
L R

L Y
C R

E R1
V S
2 2 2 2
LE LE LE LE R 2 (A) CR (B) CR ln (2)
(A) (B) (C) (D)
2 R12 2 R 22 2R1R2 2 R12 L
(C) (D) LR
Rln(2)

394,50 - Rajeev Gandhi Nagar Kota, Ph. No. : 93141-87482, 0744-2209671

Downloaded from www.iitjeephysics4u.com


IVRS No : 0744-2439051, 52, 53, www. motioniitjee.com, info@motioniitjee.com
Page # 46 ELECTROMAGNETIC INDUCTION

78. Two inductors L1 and L2 are connected in 82. In a series L–R growth circuit, if maximum
parallel and a time varying current i flows as current and maximum induced emf in an inductor
shown. The ratio of currents i1/i2 at any time t is of inductance 3mH are 2A and 6V respectively,
L1 then the time constant of the circuit is
(A) 1 ms (B) 1/3 ms
i1 (C) 1/6 ms (D) 1/2 ms
i i
83. Two coils are at fixed locations. When coil 1
i2 has no current and the current in coil 2 increases
L2 at the rate 15.0 A/s the e.m.f. in coil 1 in 25.0
(A) L1/L2 (B) L2/L1
mV, when coil 2 has no current and coil 1 has a
L21 L22 current of 3.6 A, flux linkage in coil 2 is
(C) (D)
(L1  L 2 ) 2 (L1  L 2 ) 2 (A) 16 mWb (B) 10 mWb
(C) 4.00 mWb (D) 6.00 mWb
79. Figure shows a square loop of side 0.5 m and
resistance 10. The magnetic field has a 84. Two coils of self inductance 100 mH and 400
magnitude B = 1.0T. The work done in pulling the mH are placed very close to each other. Find the
loop out of the field uniformly in 2.0 s is maximum mutual inductance between the two
B=0 when 4 A current passes through them
× × × × (A) 200 mH (B) 300 mH
× × × × v (C) 100 2 mH (D) none of these

× × × ×
(A) 3.125 × 10–3 J (B) 6.25 × 10–4 J
(C) 1.25 × 10–2 J (D) 5.0 × 10–4 J
80. In the given circuit find the ratio of i1 to i2.
Where i1 is the initial (at t = 0) current, and i2 is
steady state (at t = ) current through the
battery :
2mH

10V

(A) 1.0 (B) 0.8


(C) 1.2 (D) 1.5
81. In the circuit shown in figure, switch S is
closed at t = 0. Then :
R

R R

E
L
C
t=0

S
(A) after a long time interval potential difference
across capacitor and inductor will be equal
(B) after a long time interval charge on capacitor
will be E C.
(C) after a long time interval current in the
inductor will be E/R.
(D) after a long time interval current through
battery will be same as the current through it
initially.

394,50 - Rajeev Gandhi Nagar Kota, Ph. No. : 93141-87482, 0744-2209671

Downloaded from www.iitjeephysics4u.com IVRS No : 0744-2439051, 52, 53, www. motioniitjee.com, info@motioniitjee.com
ELECTROMAGNETIC INDUCTION Page # 47

Exercise - II
SECTION (A) : FLUX AND FARADAY’S LAWS (A) at point P is clockwise
OF ELECTROMAGNETIC INDUCTION & LENZ’S (B) at point Q is anticlockwise
LAW (C) at point Q is clockwise
1. The dimension of the ratio of magnetic flux (D) at point R is zero
and the resistance is equal to that of : 7. A conducting wire frame is placed in a magnetic
(A) induced emf (B) charge field which is directed into the paper. The magnetic
(C) inductance (D) current field is increasing at a constant rate. The
Question No. 2 to 5 (4 questions) directions of induced currents in wires AB and CD
The adjoining figure shows two different are
arrangements in which two square wire frames of × × × × × × ×
A C
same resistance are placed in a uniform constantly × × × × × × ×
decreasing magnetic field B.
× × × × × × ×
×h × × × g× B
×h × × ×g × × × × × × × × ×
× e× × × × D
× × × × × × f
f e × × × × × × ×
× × × × × × × × × × × (A) B to A and D to C (B) A to B and C to D
× × × × c × d× × d
× × × × (C) A to B and D to C (D) B to A and C to D
 c
×a × × × × × ×a × × × × 8. A bar magnet is moved along the axis of copper
L b
× × × × × × b ring placed far away from the magnet. Looking
I × × L× × ×
from the side of the magnet, an anticlockwise
II
2. The value of magnetic flux in each case is current is found to be induced in the ring. Which
given by of the following may be true ?
(A) Case I :  = (L2 +  2 ) B; Case II :  = (L2 –  2 ) B (A) The south pole faces the ring and the magnet
moves towards it.
(B) Case I :  = (L2 +  2 ) B; Case II :  = (L2 +  2 ) B (B) The north pole faces the ring and the magnet
(C) Case I :  = (L2 +  2 ) B; Case II :  = (L2 –  2 ) B moves towards it.
(C) The south pole faces the ring and the magnet
(D) Case I :  = (L +  )2 B; Case II :  = (L –  )2 B
moves away from it.
(D) The north pole faces the ring and the magnet
3. The direction of induced current in the case I is moves away from it.
(A) from a to b and from c to d
(B) from a to b and from f to e 9. Two circular coils P & Q are fixed coaxially &
(C) from b to a and from d to c carry currents I1 and I2 respectively
(D) from b to a and from e to f I1 I2
4. The direction of induced current in the case II is
(A) from a to b and from c to d
(B) from b to a and from f to e
(C) from b to a and from c to d P Q
(D) from a to b and from d to c (A) if I2 = 0 & P moves towards Q, a current in
the same direction as I1 is induced in Q
5. If I1 and I2 are the magnitudes of induced current
(B) if I1 = 0 & Q moves towards P, a current in the
in the cases I and II, respectively, then
opposite direction to that of I2 is induced in P.
(A) I1 = I2 (B) I1 > I2
(C) When I1  0 and I2  0 are in the same direction
(C) I1 < I2 (D) nothing can be said
then the two coils tend to move apart.
6. Figure shown plane figure made of a conductor (D) when I1  0 and l2  0 are in opposite directions
located in a magnetic field along the inward normal then the coils tends to move apart.
to the plane of the figure. The magnetic field
starts diminishing. Then the induced current 10. Which of the following quantities can be
P
× × × ×
written in SI units in K gm2 A–2 S–3 ?
O
(A) Resistance (B) Inductance
× × × ×
(C) Capacitance (D) Magnetic flux
× × × R×

× × × ×

394,50 - Rajeev Gandhi Nagar Kota, Ph. No. : 93141-87482, 0744-2209671

Downloaded from www.iitjeephysics4u.com


IVRS No : 0744-2439051, 52, 53, www. motioniitjee.com, info@motioniitjee.com
Page # 48 ELECTROMAGNETIC INDUCTION

SECTION (B) : E.M.F. INDUCED IN A MOVING SECTION (C) : INDUCED EMF IN ROD, RING DISC
ROD ROTATING IN A UNIFORM MAGNETIC FIELD
11. AB and CD are smooth parallel rails, separated 15. The e.m.f induced in a coil of wire, which is
by a distance l, and inclined to the horizontal at rotating in a magnetic field, does not depend on
an angle . A uniform magnetic field of magnitude (A) the angular speed of rotation
B, directed vertically upwards, exists in the region. (B) the area of the coil
EF is a conductor of mass m, carrying a current i. (C) the number of turns on the coil
For EF to be in equilibrium, (D) the resistance of the coil
B F D
Question No. 17 to 19 (3 questions)
A conducting ring of radius a is rotated about a
C  point O on its periphery as shown in the figure in
B a plane perpendicular to uniform magnetic field B
L which exists everywhere. The rotational velocity
E is .
B
 × × × × × × × ×
A × × × ×P × × × ×
(A) i must flow from E to F (B) Bil = mg tan 
× × × × × × × ×
(C) Bil = mg sin  (D) Bil = mg × × × X× × × × ×
× O× × × × × Q× ×
12. In the previous question, if B is normal to × × × × × × × ×
the plane of the rails × × × × × × × ×
(A) Bil = mg tan  (B) Bil = mg sin  × × × ×R × × × ×
(C) Bil = mg cos  16. Choose the correct statement(s) related to
(D) equilibrium cannot be reached the potential of the points P, Q and R
(A) Vp – VO > 0 and VR – V0 < 0
13. A conducting rod PQ of length L = 1.0 m is (B) Vp = VR > VO
moving with a uniform speed v = 20 m/s in a (C) VO > VP = VQ
uniform magnetic field B = 4.0 T directed into the (D) VQ – VP = VP – VO
paper. A capacitor of capacity C = 10 F is 17. choose the correct statement(s) related to
connected as shown in figure. Then the magnitude of potential differences
× × P× × ×
1 1
(A) VP – VO = Ba 2 (B) VP – VQ = Ba 2
×A × × × × 2 2
v (C) VQ – VO = 2 Ba2 (D) VP – VR = 2Ba2
B
× × × × ×
18. Choose the correct statement(s) related to
× × Q× × × the induced current in ring
(A) qA = + 800 C and qB = – 800 C (A) Current flows from Q  P  O  R  Q
(B) qA = – 800 C and qB = + 800 C (B) Current flows from Q  R  O  P  Q
(C) qA = 0 = qB (C) Current flows from QPO and from QR O
(D) charged stored in the capacitor increases (D) No current flows
exponentially with time SECTION (D) :
14. A semicircle conducting ring of radius R is SELF INDUCTANCE, MUTUAL INDUCTANCE,
placed in the xy plane, as shown in the figure. A CIRCUIT CONTAINING RESISTANCE
uniform magnetic field is set up along the x-axis. INDUCTANCE, AND L.C. COSILATIONS
No net emf, will be induced in the ring. if 19. Current growth in two L-R circuits (b) and
Y (c) as shown in figure (a). Let L1, L2, R1 and R2 be
the corresponding values in two circuits. Then
B
i
R
(b)
X (c) R1
L1
Z
(A) it moves along the x-axis t
(B) it moves along the y-axis (a) S
(C) it moves along the z-axis V
(b)
(D) it remains stationary

394,50 - Rajeev Gandhi Nagar Kota, Ph. No. : 93141-87482, 0744-2209671

Downloaded from www.iitjeephysics4u.com


IVRS No : 0744-2439051, 52, 53, www. motioniitjee.com, info@motioniitjee.com
ELECTROMAGNETIC INDUCTION Page # 49

C1
L2 R2

L S

V S
(c) C2
(A) R1 > R2 (B) R1 = R2 (A) maximum current through inductor equals I0/2.
(C) L1 > L2 (D) L1 < L2 (B) maximum current through inductor equals
C1I0
20. A circuit element is placed in a closed box. At C1  C 2
time t = 0, constant current generator supplying C1 I0 LC1
a current of 1 amp, is connected across the box. (C) maximum charge on C1 =
Potential difference across the box varies C1  C 2
according to graph shown in figure. The element L
in the box is : (D) maximum charge on C1 = I0 C1
V(volts) C1  C 2
25. For L - R circuit, the time constant is equal to
8 (A) twice the ratio of the energy stored in the
magnetic field to the rate of the dissipation of
2 energy in the resistance.
(B) the ratio of the energy stored in the magnetic
field to the rate of dissipation of energy in the
t(s) resistance.
3
(A) resistance of 2 (B) battery of emf 6 V (C) half of the ratio of the energy stored in the
(C) inductance of 2H (D) capacitance of 0.5F magnetic field to the rate of dissipation of energy
in the resistance.
21. The symbols L, C, R represent inductance, (D) square of the ratio of the energy stored in
capacitance and resistance respec tively. the magnetic field to the rate of dissipation energy
Dimension of frequency are given by the in the resistance.
combination Sol.
1 26. An inductor L, a resistance R and two identical
(A) 1/RC (B) R / L (C) (D) C / L
LC bulbs B1 and B2 are connected to a battery through
a switch S as shown in the figure. The resistance
22. An LR circuit with a battery is connected at t of coil having inductance L is also R. Which of
= 0. Which of the following quantities is not zero the following statement gives the correct
just after the circuit description of the happenings when the switch S
(A) current in the circuit is closed ?
(B) magnetic field energy in the inductor L B1
(C) power delivered by the battery
(D) emf induced in the inductor R

23. The switches in figures (a) and (b) are


closed at t = 0 E S
C R L (A) The bulb B2 lights up earlier than B1 and finally
R
) both the bulbs shine equally bright.
(B) B1 light up earlier and finally both the bulbs
acquire equal brightness
(C) B2 lights up earlier and finally B1 shines brighter
than B2.
(D) B 1 and B 2 light up together with equal
E E
(a) (b) brightness all the time.
(A) The charge on C just after t = 0 is EC.
(B) The charge on C long after t = 0 is EC. 27. In figure, the switch S is closed so that a
(C) The current in L just after t = 0 is E/R. current flows in the iron-core inductor which has
(D) The current in L long after t = 0 is E/R. inductance L and the resistance R. When the
switch is opened, a spark is obtained in it at the
24. At a moment (t = 0) when charge on capacitor
C1 is zero, the switch is closed. If I0 be the current contacts. The spark is due to
through inductor at that instant, for t > 0,

394,50 - Rajeev Gandhi Nagar Kota, Ph. No. : 93141-87482, 0744-2209671

Downloaded from www.iitjeephysics4u.com


IVRS No : 0744-2439051, 52, 53, www. motioniitjee.com, info@motioniitjee.com
Page # 50 ELECTROMAGNETIC INDUCTION

L
31 Two different coils have self inductance 8mH
R and 2mH. The current in one coil is increased at
a constant rate. The current in the second coil is
S also increased at the same constant. At a certain
B instant of time, the power given to the two coils
(A) a slow flux change in L is the same. At that time the current, the induced
(B) a sudden increase in the emf of the battery B voltage and the energy stored in the first coil are
(C) a rapid flux change in L I1, V1 and W1 respectively. Corresponding values
(D) a rapid flux change in R for the second coil at the same instant are I2, v2
and W2 respectively. Then :
28. In figure, a lamp P is in series with an iron-
I1 1 I1
core inductor L. When the switch S is closed, the (A) I  4 (B) I  4
brightness of the lamp rises relatively slowly to 2 2

its full brightness than it would do without the W2 V2 1


inductor. This is due to (C) W  4 (D) V  4
P L 1 1

S
B
(A) the low resistance of P
(B) the induced-emf in L
(C) the low resistance of L
(D) the high voltage of the battery B

29. Two coil A and B have coefficient of mutual


inductance M = 2H. The magnetic flux passing
through coil A changes by 4 Weber is 10 seconds
due to the change in current in B. Then
(A) change in current in B in this time interval is
0.5 A
(B) the change in current in B in this time interval is
2A
(C) the change in current in B in this time interval is
8A
(D) a change in current of 1A in coil A will produce
a change in flux passing through B by 4 Weber.
30. An inductance L, resistance R, battery B and
switch S are connected in series. Voltmeters VL
and VR are connected across L and R respectively.
When switch is closed :
VL VR

L R

B S
(A) The initial reading in VL will be greater than in
VR
(B) The initial reading in VL will be lesser than VR
(C) The initial readings in VL and VR will be the
same.
(D) The reading in VL will be decreasing as time
increases.

394,50 - Rajeev Gandhi Nagar Kota, Ph. No. : 93141-87482, 0744-2209671

Downloaded from www.iitjeephysics4u.com


IVRS No : 0744-2439051, 52, 53, www. motioniitjee.com, info@motioniitjee.com
ELECTROMAGNETIC INDUCTION Page # 51

Exercise - III (SUBJECTIVE PROBLEMS)

1. The horizontal component of the earth’s circulating the inner coil.


magnetic field at a place is 3 × 10–4 T and the dip
is tan–1(4/3). A metal rod of length 0.25 m placed b
a
in the north-south position is moved at a constant
speed of 10 cm/s towards the east. Find the e.m.f
induced in the rod.
2. A wire forming one cycle of sine curve is moved 7. A horizontal wire is free to slide on the vertical
 rails of a conducting frame as shown in figure.
in x-y plane with velocity V  V i  V j . There exist
x y
 The wire has a mass m and length l and the
a magnetic field B  –B 0k . Find the motional emf resistance of the circuit is R. If a uniform magnetic
develop across the endsPQ of wire. field B is directed perpendicular to the frame, then
y
B find the terminal speed of the wire as it falls under
× × × × × × the force of gravity.
× × × × × ×
× P× × × × Q× l
× × × × × × X
× × × × × × × × × × × m×
× × × × × × × × × × × ×
×z × × × × × × ×B × × × ×
× × × × × × × × × × × ×
× × × × × ×
× × × × × ×
3. A conducting circular loop is placed in a uniform × × × × × ×
× × ×R × × ×
magnetic f ield of 0.02 T, with its plane
perpendicular to the field. If the radius of the 8. A metal rod of resistance 20 is fixed along a
loop starts shrinking at a constant rate of 1.0 diameter of a conducting ring of radius 0.1 m and
mm/s, then find the emf induced in the loop, at lies on x-y plane. There is a magnetic field
the instant when the radius is 4 cm. 
B  ( 50 T)k . The ring rotates with an angular
L velocity  = 20 rad/sec about its axis. An external
4. Find the dimension of the quantity , where
RCV resistance of 10 is connected across the centre
symbols have usual meaning. of the ring and rim. Find the current through
external resistance.
5. A rectangular loop with a sliding connector of
length l = 1.0 m is situated in a uniform magnetic 9. There exists a uniform cylindrically symmetric
field B = 2T perpendicular to the plane of loop. magnetic field directed along the axis of a cylinder
Resistance of connector is r = 2. Two resistances but varying with time as B = kt. If an electron is
of 6 and 3 are connected as shown in figure. released from rest in this field at a distance of ‘r’
Find the external force required to keep the from the axis of cylinder, its acceleration, just
connector moving with a constant velocity v = 2m/ after it is released would be (e and m are the
s. electronic charge and mass respectively)

B 10. An emf of 15 volt is applied in a circuit
6 3 containing 5 H inductance and 10  resistance.
Find the ratio of the currents at time t =  and t
= 1 second.

6. Two concentric and coplanar circular coils have 11. A uniform magnetic field of 0.08 T is directed
radii a and b (>> a) as shown in figure. Resistance into the plane of the page and perpendicular to it
of the inner coil is R. Current in the outer coil is as shown in the figure. A wire loop in the plane of
increased from 0 to i, then find the total charge the page has constant area 0.010 m 2. The

394,50 - Rajeev Gandhi Nagar Kota, Ph. No. : 93141-87482, 0744-2209671

Downloaded from www.iitjeephysics4u.com


IVRS No : 0744-2439051, 52, 53, www. motioniitjee.com, info@motioniitjee.com
Page # 52 ELECTROMAGNETIC INDUCTION

magnitude of magnetic field decrease at a 13. Two coils, 1 & 2, have a mutual inductance =
constant rate of 3.0 × 10 –4 Ts –1 . Find the M and resistance R each. A current flows in coil
magnitude and direction of the induced emf in 1, which varies with time as : I1 = kt2, where K is
the loop. a constant and ‘t’ is time. Find the total charge
× × × × × × that has flown through coil 2, between t = 0 and
× × × × × ×
× × × × × × t = T.
× × × × × ×
× × × × × × 14. A charged ring of mass m = 50 gm, charge 2
× × ×B× × × coulomb and radius R = 2m is placed on a smooth
× × × × × ×
horizontal surface. A magnetic field varying with
× × × × × ×
time at a rate of (0.2 t) Tesla/sec is applied on
12. In the circuit shown in figure switch S is closed to the ring in a direction normal to the surface of
at time t = 0. Find the charge which passes ring. Find the angular speed attained in a time t1
through the battery in one time constant. = 10 sec.

15. A uniform but time varying magnetic field B =


L R Kt – C ; (0  t  C/K), where K and C are constant
and t is time, is applied perpendicular to the plane
of the circular loop of radius ‘a’ and resistance R.
E S Find the total charge that will pass around the
loop.

394,50 - Rajeev Gandhi Nagar Kota, Ph. No. : 93141-87482, 0744-2209671

Downloaded from www.iitjeephysics4u.com


IVRS No : 0744-2439051, 52, 53, www. motioniitjee.com, info@motioniitjee.com
ELECTROMAGNETIC INDUCTION Page # 53

Exercise - IV (TOUGH SUBJECTIVE PROBLEMS)


1. Two straight conducting rails from a right angle 4. Consider the possibility of a new design for an
where their ends are joined. A conducting bar electric train. The engine is driven by the force
contact with the rails starts at vertex at the due to the vertical component of the earths wheel
time t = 0 & moves symmetrically with a constant & then back to the source via the other rail.
velocity of 5.2 m/s to the right as shown in figure. (i) What current is needed to provide a modest
A 0.35 T magnetic field points out of the page. 10 – KN force ? Take the vertical component of
Calculate : the earth’s field be 10 T & the length of axle to be
(i) The flux through the triangle by the rails & bar 3.0 m.
at t = 3.0 s. (ii) How much power would be lost for each  of
(ii) The emf around the triangle at that time. resistivity in the rails?
(iii) In what manner does the emf around the (iii) is such a train realistic?
triangle vary with time.
5. A square wire loop with 2 m sides in
B
perpendicular to a uniform magnetic field, with
5.2 m/s half the area of the loop in the field. The loop
90°
contains a 20 V battery with negligible internal
resistance. If the magnitude of the field varies
with time according to B = 0.042 – 0.87 t, with B
in tesla & t in sec.
2. Two long parallel rails, a distance l apart and (i) What is the total emf in the circuit ?
each having a resistance . per unit length are (ii) What is the direction of the current through
joined at one end by a resistance R. A perfectly the battery ?
conducting rod MN of mass m is free to slide along
the rails without friction. There is a uniform
magnetic field of induction B normal to the plane
of the paper and directed into the paper. A variable
force F is applied to the rod MN such that, as the
rod moves, a constant current i flows through R. 6. A rectangular loop of dimensions l & w and
Find the velocity of the rod and the applied force resistance R moves with constant velocity V to
F as function of the distance x of the rod from R the right as shown in the figure. It continues to
move with same speed through a region containing
M
a uniform magnetic field B directed into the plane
 of the paper & extending a distance 3 W. Sketch
B
R the flux, induced emf & external force acting on
d
the loop as a function of the distance.
x
3w
N
× × × × ×
v × × × × B×
3. A wire is bent into 3 circular segments of radius × × × × ×
s× ×v × × ×
r = 10 cm as shown in figure. Each segment is a
l × × × × ×
quadrant of a circle, ab lying in the xy plane, bc × × × × ×
lying in the yz plane & ca lying in the zx plane. w × × × × ×
(i) if a magnetic field B points in the positive × × × × ×
x direction, what is the magnitude of the emf
developed in the wire, when B increases at the 8. A square loop of side ‘a’ & resistance R moves
rate of 3 mT/s? with a uniform velocity v away from a long wire
z
that carries current I as shown in the figure. The
c loop is moved away from the wire with side AB
always parallel to the wire. Initially, distance
r r between the side AB of the loop & wire is ‘a’. Find
b
y the work done when the loop is moved through
a distance ‘a’ from the initial position
x
(ii) what is the direction of the current in the
segment bc.

394,50 - Rajeev Gandhi Nagar Kota, Ph. No. : 93141-87482, 0744-2209671

Downloaded from www.iitjeephysics4u.com


IVRS No : 0744-2439051, 52, 53, www. motioniitjee.com, info@motioniitjee.com
Page # 54 ELECTROMAGNETIC INDUCTION

portion ABC, CDA and AMC are R1, R2 and R3


a
B C respectively. What current will be shown by meter
I
a M ? The magnetic field is concentrated near the
v
axis of the circular conductor.
.
A D
A
M
B
××
9. Two long parallel conducting horizontal rails ×× C
are connected by a conducting wire at one end.
D
A uniform magnetic field B exists in the region of
space. A light uniform ring of diameter d which is
practically equal to separation between the rails, 13. In the circuit shown in the figure the switched
is placed over the rails as shown in the figure. If S1 and S2 are closed at time t = 0. After time t =
resistance of ring is  per unit length, calculate (0.1) ln 2 sec, switch S2 is opened. Find the
the force required to pull the ring with uniform current in the circuit at time t = (0.2) ln 2 sec.
velocity v.
S1

B 40 S2
d F
10
100V
1H

10. A long straight wire is arranged along the


symmetry axis of a toroidal coil of rectangular 14. Find the values of i1 and i2
cross-section, whose dimensions are given in the (i) immediately after the switch S is closed.
figure. The number of turns on the coil is N, and (ii) long time later, with S closed.
relative permeability of the surrounding medium is (iii) immediately after S is open.
unity. Find the amplitude of the emf induced in (iv) long time after S is opened
this coil, if the current i = im cos t flows along 15. Suppose the emf of the battery, the circuit
the straight wire. shown varies with time t so the current is given by
b
i(t) = 3 + 5t, where i is in amperes & t is in seconds.
Take R = 4, L = 6H & find an expression for the
a battery emf as function of time.
h
R i(t)

 L

11. A uniform magnetic field B fills a cylindrical
volumes of radius R. A metal rod CD of length l is
placed inside the cylinder along a chord of the 16. A rectangular frame ABCD made of a uniform
circular cross-section as shown in the figure. If metal wire has a straight connection between E
the magnitude of magnetic field increases in the & F made of the same wire as shown in the figure.
direction of field at a constant rate dB/dt, find AEFD is a square of side 1 m & EB = FC = 0.5 m.
the magnitude and direction of the EMF induced The entire circuit is placed in a steadily increasing
in the rod. uniform magnetic field directed into the place of
 the paper & normal to it. The rate of change of
× × × × B the magnetic field is 1 T/s, the resistance per
× × × × × ×
×× × × × × ××
unit length of the wire is 1 /m. Find the current
b ×× × × × × ×× in segments AE, BE & EF.
×× × × × × ××
C × × × × × × D A E B
× × × ×
×× × × ×
×× × × ×

12. A variable magnetic field creates a constant ××B × × ×
×
emf E in a conductor ABCDA. The resistances of
D F C
394,50 - Rajeev Gandhi Nagar Kota, Ph. No. : 93141-87482, 0744-2209671

Downloaded from www.iitjeephysics4u.comIVRS No : 0744-2439051, 52, 53, www. motioniitjee.com, info@motioniitjee.com


ELECTROMAGNETIC INDUCTION Page # 55

17. Two parallel vertical metallic rails AB & CD are 19. A pair of parallel horizontal conducting rails
separated by 1m. They are connected at the of negligible resistance shorted at one end is fixed
two ends by resistance R1 & R2 as shown in the on a table, The distance between the rails is L. A
figure. A horizontally metallic bar L of mass 0.2 conducting massless rod of resistance R can slide
kg slides without friction, vertically down the rails on the rails frictionlessly. The rod is tied to a
under the action of gravity. There is a uniform massless string which passes over a pulley fixed
horizontal magnetic field of 0.6 T perpendicular to the edge of the table. A mass m, tied to the
to the plane of the rails, it is observed that when other end of the string hangs vertically. A
the terminal velocity is attained, the power constant magnetic field B exists perpendicular to
dissipated in R 1 & R 2 are 0.76 W & 1.2 W the table. If the system is released from rest,
respectively. Find the terminal velocity of bar L & calculate.
value R1 & R2. (i) the terminal velocity achieved by the rod.
(ii) the acceleration of the mass at the instant
A C when the velocity of the rod is half the terminal
L velocity. [JEE ‘97, 5]

R2
B D

18. A metal rod OA of mass m & length r is kept


rotating with a constant angular speed  in a m
vertical plane about a horizontal axis at the end
O. The free end A is arranged to slide without
friction along a fixed conducting circular ring in
20. A magnetic field B = (B0 y/a) k is into the
the same plane as that of rotation. A uniform &
 plane of paper in the +z direction. B0 and a are
constant. magnetic induction B is applied positive constants. A square loop EFGH of side a,
perpendicular & into the plane of rotation as shown mass m and resistance R, in x-y plane, starts
in figure. An inductor L and an external resistance falling under the influence of gravity. Note the
R are connected through a switch S between the directions of x and y in the figure. Find
point O & a point C on the ring to form an electrical [JEE ‘99]
circuit. Neglect the resistance of the ring and
O x
the rod. Initially, the switch is open.
  
× × × y× × E F
× × × × ×   
× × × × A× G H g
s × × × × ×   
× × ×O × × x
R y
× × × × ×
× × ×C × × (a) the induced current in the loop and indicate
L
its direction,
(b) the total Lorentz force acting on the loop
(a) What is the induced emf across the terminals and indicate its direction.
of the switch ? (c) an expression for the speed of the loop, v(t)
(b) (i) Obtain an expression for the current as a and its terminal value.
function of time after switch S is closed.
(ii) Obtain the time dependence of the torque
required to maintain the constant angular speed,
given that the rod OA was along the positive X-
axis at t = 0.

394,50 - Rajeev Gandhi Nagar Kota, Ph. No. : 93141-87482, 0744-2209671

Downloaded from www.iitjeephysics4u.com


IVRS No : 0744-2439051, 52, 53, www. motioniitjee.com, info@motioniitjee.com
Page # 56 ELECTROMAGNETIC INDUCTION

Exercise - V (JEE PROBLEMS)

1. Two circular coils can be arranged in any of of turns were to be quadrupled and the wire radius
halved, the electrical power dissipated would be
the three situations shown in the figure. Their
[JEE 2002 (Scr, 3)
mutual inductance will be (A) halved (B) the same
(C) doubled (D) quadrupled

5. A square loop of side ‘a’ with a capacitor of


(a) (b) (c) capacitance C is located between two current
(A) maximum in situation (a) carrying long parallel wires as shown. The value
(B) maximum in situation (b) of I in the is given as I = I0 sin t.
(C) maximum in situation (c)
(D) the same in all situations JEE’2001,(Scr)]
a a
2. An inductor of inductance L = 400 mH and I I
resistors of resistances R1 = 2 and R2 = 2 are a
connected to a battery of e.m.f E = 12V as shown
in the figure. The internal resistance of the battery
is negligible. The switch S is closed at time t = 0. (a) calculate maximum current in the square loop.
What is the potential drop across L as a function (b) Draw a graph between charge on the lower
of time ? After the steady state is reached, the plate of the capacitor v/s time. [JEE 2003]
switch is opened. What is the direction and the
magnitude of current through R1 as a function of 6. The variation of induced emf () with time (t)
time? [JEE, 2001] in a coil if a short bar magnet is moved along its
axis with a constant velocity is best represented
E L as

R1
S R2
 

3. As shown in the figure, P and Q are two coaxial (A) (B) t


conducting loops separated by some distance.
t
When the switch S is closed, a clockwise current

Ip flows in P (as seen by E) and an induced current 
IQI flows in Q The switch remains closed for a long
time. When S is opened, a current IQ2 flows in Q.
Then the directions of IQI and IQ2 (as seen by E) (C) t (D) t
are :
P Q [JEE 2004 (Scr)]

E
7. An infinitely long cylindrical conducting rod is
kept along +Z direction. A constant magnetic field
S is also present in +Z direction. Then current
Battery induced will be
(A) respectively clockwise and anti-clockwise (A) 0 (B) along +Z direction
(B) both clockwise (C) along clockwise as seen from +Z
(C) both anti-clockwise (D) along anticlockwise as seen from +Z
(D) respectively anti-clockwise and clockwise [JEE’ 2005 (Scr)]
[JEE 2002 (Scr), 3]
8. A long solenoid of radius a and number of turns
4. A short-circuited coil is placed in a time varying per unit length n is enclosed by cylindrical shell
magnetic field. Electrical power is dissipated due of radius R, thickness d (d<<R) and length L. A
to the current induced in the coil. If the number
394,50 - Rajeev Gandhi Nagar Kota, Ph. No. : 93141-87482, 0744-2209671

Downloaded from www.iitjeephysics4u.com


IVRS No : 0744-2439051, 52, 53, www. motioniitjee.com, info@motioniitjee.com
ELECTROMAGNETIC INDUCTION Page # 57

variable current i = i0 sint flows through the half of the heat dissipated when the capacitor is
coil. If the resistivity of the material of cylindrical fully charged.
shell is , find the induced current in the shell. (D) after time interval 2, charge on the capacitor
R [JEE 2005] is CV(1 – e–1) [JEE 2006]
a

d 11. After the capacitor gets fully charged, S1 is


opened and S2 is closed so that the inductor is
connected in series with the capacitor. Then,
(A) at t = 0, energy stored in the circuit is purely
L in the form of magnetic energy
(B) at any time t > 0, current in the circuit is in
the same direction.
(C) at t > 0, there is no exchange of energy
between the inductor and capacitor
(D) at any time t > 0, instantaneous current in
C
9. In the given diagram, a line of force of a the circuit may V [JEE 2006]
L
particular force field is shown. Out of the following
options, it can never represent 12. If the total charge stored in the LC circuit is
Q0, then for t  0
(A) the charge on the capacitor is Q =Q0 cos
 t 
  
2 LC 
(B) the charge on the capacitor is Q =Q0cos
(A) an electrostatic field
(B) a magnetostatic field  t 
 – 
(C) a gravitational field of a mass at rest 2 LC 
(D) an induced electric field
d2 Q
[JEE 2006] (C) the charge on the capacitor is Q = – LC
dt 2
COMPREHENSION - I 1 d2 Q
(D) the charge on the capacitor is Q = –
The capacitor of capacitance C can be charged LC dt 2
(with the help of a resistance R) by a voltage [JEE 2006]
source V, by closing switch S1 while keeping switch
13. Match the following Columns
S2 open. The capacitor can be connected in series Column I
with an inductor ‘L’ by closing switch S2 and Column II
opening S1 (A) Dielectric ring uniformly charged ( P )
v
Time independent electrostatic field out of
system
R S1 (B) Dielectric ring uniformly charged (Q )
C Magnetic field
rotating with angular velocity
S2 (C) Constant current i0 in ring ( R )
L Induced electric field
(D) Current i = i0 cos t in ring ( S )
Magnetic moment
10. Initially, the capacitor was uncharged. Now,
switch S1 is closed and S2 is kept open. If time [JEE 2006]
constant of this circuit is , then
(A) after time interval , charge on the capacitor
is CV/2
(B) after time interval 2, charge on the capacitor
is CV(1–e2)
(C) the work done by the voltage source will be

394,50 - Rajeev Gandhi Nagar Kota, Ph. No. : 93141-87482, 0744-2209671

Downloaded from www.iitjeephysics4u.com


IVRS No : 0744-2439051, 52, 53, www. motioniitjee.com, info@motioniitjee.com
Page # 58 ELECTROMAGNETIC INDUCTION
14. Statement-I
A vertical iron rod has a coil of wire wound over it at the bottom end. An alternating current flows in
the coil. The rod goes through a conducting ring as shown in the figure The ring can float at a
certain height above the coil

because
Statement - II
In the above situation, a current is induced in the ring which interacts with the horizontal component
of the magnetic field to produce an average force in the upward direction.
(A) Statement-1 is True, Statement-2 is True; Statement-2 is a correct explanation for Statement-1
(B) Statement-1 is True, Statement-2 is True; Statement-2 is NOT a correct explanation for Statement-
1
(C) Statement-1 is True, Statement-2 is False
(D) Statement-1 is False, Statement-2 is True [JEE 2007]

15. This section contains 2 questions. Each questions contains statements given in two columns,
which have to be matched. The statements in Column I are labelled A, B, C and D, while the
statements in Column II are labelled p, q, r, s and t. Any given statement in Column I can have
correct matching with one or more statement(s) in Column II. The appropriate bubbles corresponding
to the answers to these questions have to be darkened as illustrated in the following example :
If the correct matches are A – p, s and t; B – q and r ; C – p and q; and D – s and t; then the correct
darkening of bubbles will look like the following.
Column II gives certain systems undergoing a process. Column I suggests changes in some of the
parameters related to the system. Match the statements in Column I to the appropriate process(es)
from Column II. [JEE 2009]
Column I Column II
(A) The energy of the system is (P) System: A capacitor initially uncharged Increased
Increased Process: It is connected to a battery
(B) Mechanical energy is provided to (Q) System : A gas in an adiabatic container fitted with
anthe system, which is converted into adiabatic piston
energy of random motion of its parts Process : The gas is compressed by pushing the piston
(C) Internal energy of the system is (R) System : a gas in a rigid container
converted into its mechanical energy. Process : The gas gets cooled due to colder
atmosphere surrounding it
(D) Mass of the system is decreased (S) System : A heavy nucleus initially at rest
Process : The nucleus fissions into two fragments of
nearly equal masses and some
neutrons are emitted
(T) System : A resistive wire loop
Process : The loop is placed in a time varying magnetic
field perpendicular to its plane.

16. Two metallic rings A and B, identical in shape and size but having different resistivities A and B
, are kept on top of two identical solenoids as shown in the figure. When current I is switched on in
both the solenoids in identical manner, the rings A and B jump to heights hA and hB , respectively, with
hA > hB . The possible relation(s) between their resistivities and their masses mA and mB is (are)
[JEE 2009]

394,50 - Rajeev Gandhi Nagar Kota, Ph. No. : 93141-87482, 0744-2209671

Downloaded from www.iitjeephysics4u.com


IVRS No : 0744-2439051, 52, 53, www. motioniitjee.com, info@motioniitjee.com
ELECTROMAGNETIC INDUCTION Page # 59

(A) A > B and mA = mB


(B) A < B and mA = mB
(C) A > B and mA > mB
(D) A < B and mA < mB

17. The figure shows certain wire segments joined (A) (B)
together to form a coplanar loop. The loop is
placed in a perpendicular magnetic field in the
direction going into the plane of the figure. The
magnitude of the field increases with time. I1
and I2 are the currents in the segments ab and
cd. Then, [JEE 2009]

(C) (D)

20. A current carrying infinitely long wire is kept


(A) I1 > I2 (B) I1 < I2 along the diameter of a circular wire loop, without
(C) I1 is in the direction ba and I2 is in the direction touching it. The correct statement(s) is(are)
cd [JEE 2012]
(D) I1 is in the direction ab and I2 is in the direction (A) The emf induced in the loop is zero if the
dc
current is constant.
(B) The emf induced in the loop is finite if the
18. A long circular tube of length 10m and radius
current is constant.
0.3 m carries a current l along its curved surface
(C) The emf induced in the loop is zero if the
as shown. A wire-loop of resistance 0.005 ohm
current decreases at a steady rate.
and of radius 0.1 m is placed inside the tube with
(D) The emf induced in the loop is finite if the
its axis coinciding with the axis of the tube. The
current decreases at a steady rate.
current varies as l = l0 cos (300 t) where I0 is
21. A circular wire loop of radius R is placed in
constant. If the magnetic moment of the loop is
the x-y plane centered at the origin O. A square
N 0 I0 sin(300t), then 'N' is [JEE 2011]
loop of side a(a<<R) having two turns is placed
with its center at z = 3 R along the axis of the
circular wire loop, as shown in figure. The plane
o
of the square loop makes an angle of 45 with
respect to the z-axis. If the mutual inductance
 0 a2
between the loops is given by p / 2 , then value
2 R
of p is [JEE 2012]
45o

19. Which of the field patterns given below is


3 R
valid for electric field as well as for magnetic field?
[JEE 2011]

394,50 - Rajeev Gandhi Nagar Kota, Ph. No. : 93141-87482, 0744-2209671

Downloaded from www.iitjeephysics4u.com


IVRS No : 0744-2439051, 52, 53, www. motioniitjee.com, info@motioniitjee.com
Page # 60 ELECTROMAGNETIC INDUCTION

ANSWER KEY EXERCISE - I


1. C 2. B 3. C 4. A 5. B 6. A 7. A

8. C 9. C 10. C 11. A 12. A 13. D 14. D

15. B 16. C 17. A 18. B 19. D 20. B 21. A

22. A 23. A 24. D 25. B 26. C 27. D 28. A

29. A 30. C 31. B 32. D 33. D 34. A 35. B

36. A 37. C 38. A 39. D 40. D 41. B 42. C

43. A 44. C 45. A 46. D 47 B 48. D 49. A

50. C 51. C 52. B 53. A 54. A 55. C 56. B

57. A 58. C 59. A 60. C 61. A 62. A 63. A

64. B 65. B 66. C 67. A 68. B 69. A 70. D

71. C 72. A 73. B 74. C 75. D 76. D 77. B

78. B 79. A 80. B 81. D 82. A 83. D 84. A

ANSWER KEY EXERCISE - II

1. B 2. C 3. C 4. B 5. B 6. A,C,D 7. A

8. B,C 9. B,D 10. A 11. A,B 12. B 13. A 14. A,B,C,D

15. D 16. B,D 17. C 18. D 19. B,D 20. D 21. A,B,C

22. D 23. B,D 24. D 25. A 26. A 27. C 28. B

29. B 30. A,D 31. A,C,D

ANSWER KEY EXERCISE - III

 0 ia2 
1. 10 V 2.  VyB0 Q.3 5.0 V 4. I–1 5. 2 N 6.
2Rb
mgR 1
7. 2 2 8. A
B l 3
erk
9. directed along tangent to the circle of radius r, whose centre lies on the axis of cylinder..
2m
2
e EL
10. 11. 3 V, clockwise 12. 13. kMT2/R 14. 200 rad/sec
2
e –1 eR 2
15. C  a2 / R

394,50 - Rajeev Gandhi Nagar Kota, Ph. No. : 93141-87482, 0744-2209671

Downloaded from www.iitjeephysics4u.com


IVRS No : 0744-2439051, 52, 53, www. motioniitjee.com, info@motioniitjee.com
ELECTROMAGNETIC INDUCTION Page # 61

ANSWER KEY EXERCISE - IV

I(R  2x) 2I2 m(R  2x)


1. (i) 85.22 Tm2, (ii) 56.8 V; (iii) linearly 2. , + BId 3. (i) 2.4 × 10–5 V
Bd B 2 d2
(ii) from c to b
4. (i) 3.3 × 108 A, (ii) 1.1 × 1017 W, (iii) totally unrealistic

E w s
3w 4w

 0 ab
5. 21.74 V, anticlockwise 6. s 7.  = ILln
2 a
F s

2 2
 0 I2 a 2 V  2 2 3   0 I2aV  1 3 4B 2 vd  0 himN b
8. 2
  n
 3a a 4  = 2  3  n 4  9. 10. ln 11.
4 R   2 R    2 a

l dB l2 ER1
R2 – 12. R R  R R  R R 13. 67/32 A
2 dt 4 1 2 2 3 3 1

14. (i) i1 = i2 = 10/3 A, (ii) i1 = 50/11 A; i2 = 30/11 A, (iii) i1 = 0, i2 = 20/11 A, (iv) i1 = i2 =


0
7 3 1
15. 42 + 20t volt 16. IEA = A ; IBE = A ; IFE = A 17. V = 1 ms–1, R1 = 0.47
22 11 22

1 Br 2 |1 – e –Rt /L | mgr B 2r 4


, R2 = 0.30  18. (a) E = Br 2 (b) (i) I = , (ii)  = cost + (1 – e–Rt/L)
2 2R 2 4R
mgR g
19. Vterminal = 2 2 ; (ii)
B l 2
B 0 av
20. (a) i = in anticlockwise direction, v = velocity at time t, (b) Fnett = B02a2V/R, (c) V =
R

 B 20 a 2 t 
mgR 
1 – e mR 
B 20 a 2  

394,50 - Rajeev Gandhi Nagar Kota, Ph. No. : 93141-87482, 0744-2209671

Downloaded from www.iitjeephysics4u.com


IVRS No : 0744-2439051, 52, 53, www. motioniitjee.com, info@motioniitjee.com
Page # 62 ELECTROMAGNETIC INDUCTION

ANSWER KEY EXERCISE - V

1. A 2. 12 e–5t, 6e–10t 3. D 4. B

Q0

 0a
5. (a) Imax = Cl0  2 ln2 , (b) t 6. B

–Q0

(  0 ni0  cos t)a2 (Ld)


7. A 8. I= 9. A,C 10. B 11. D
2R
12. C 13. (A) P; (B) P, Q, S; (C) Q, S; (D) Q, R,S 14. A
15. (A)  P,Q,S,T ; (B)  Q ; (C)  S ; (D) S
16. B,D 17. D 18. 0006 19. C 20. A,C 21. 7

394,50 - Rajeev Gandhi Nagar Kota, Ph. No. : 93141-87482, 0744-2209671

Downloaded from www.iitjeephysics4u.com


IVRS No : 0744-2439051, 52, 53, www. motioniitjee.com, info@motioniitjee.com

You might also like